Final, Winter 2024

University of British Columbia, MATH 101 A

View Exam PDF

View Original Exam Source

Question 1
Max Marks: 3

Find G(x) G'(x) , given:

G(x)=1x35+t1+sin2(t)dt G(x) = \int_{1}^{x^3} \frac{5+t}{1+\sin^2(t)} dt
Fundamental Theorem of Calculus
Differentiation: general

Tag:

A comprehensive "Filter by Tags" search feature is available on the course homepage, allowing you to find all exercises with this tag.

Prerequisites for this Exercise

No prerequisite skills selected yet.

Find G(x) G'(x) , given:

G(x)=1x35+t1+sin2(t)dt G(x) = \int_{1}^{x^3} \frac{5+t}{1+\sin^2(t)} dt

Step 1: Identify the problem


We need to find the derivative G(x) G'(x) of a function defined by a definite integral with a variable upper limit.

Step 2: Apply the Fundamental Theorem of Calculus


Using the Fundamental Theorem of Calculus:

If G(x)=ag(x)f(t)dt G(x) = \int_{a}^{g(x)} f(t) dt , then G(x)=f(g(x))g(x) G'(x) = f(g(x)) \cdot g'(x)

In our case:
- g(x)=x3 g(x) = x^3 (the upper limit of integration)
- f(t)=5+t1+sin2(t) f(t) = \frac{5+t}{1+\sin^2(t)} (the integrand)
- g(x)=3x2 g'(x) = 3x^2 (the derivative of the upper limit)

Step 3: Substitute into the formula



G(x)=f(g(x))g(x)=f(x3)3x2=5+x31+sin2(x3)3x2 G'(x) = f(g(x)) \cdot g'(x) = f(x^3) \cdot 3x^2 = \frac{5+x^3}{1+\sin^2(x^3)} \cdot 3x^2


Step 4: Simplify the expression



G(x)=3x2(5+x3)1+sin2(x3)=15x2+3x51+sin2(x3) G'(x) = \frac{3x^2(5+x^3)}{1+\sin^2(x^3)} = \frac{15x^2+3x^5}{1+\sin^2(x^3)}


Therefore:

G(x)=15x2+3x51+sin2(x3) \boxed{G'(x) = \frac{15x^2+3x^5}{1+\sin^2(x^3)}}

Prof's perspective


Your professor designed this problem to test your understanding of the Fundamental Theorem of Calculus with variable limits.
G(x)=15x2+3x51+sin2(x3) \boxed{G'(x) = \frac{15x^2+3x^5}{1+\sin^2(x^3)}}
Question 2
Max Marks: 3

Find the area of one complete positive region bounded by the curve y=2cos(x) y = 2\cos(x) and the x-axis.
integrals
Area: General

Tag:

A comprehensive "Filter by Tags" search feature is available on the course homepage, allowing you to find all exercises with this tag.

Prerequisites for this Exercise

No prerequisite skills selected yet.

Find the area of one complete positive region bounded by the curve y=2cos(x) y = 2\cos(x) and the x-axis.

Step 1: Understand the region we're finding


We need to find the area of one complete positive region bounded by:
- The curve y=2cos(x) y = 2\cos(x)
- The x-axis where y=0 y = 0

Step 2: Find the points of intersection with the x-axis


At the x-axis, y=0 y = 0 :

0=2cos(x) 0 = 2\cos(x)
cos(x)=0 \cos(x) = 0

This occurs at x=π2+nπ x = \frac{\pi}{2} + n\pi for integer values of n n .

Since we want one complete positive region, we need to identify where the function is positive. The cosine function is positive in the interval (π2,π2)+2nπ (-\frac{\pi}{2}, \frac{\pi}{2}) + 2n\pi for integer values of n n .

For simplicity, let's choose the interval [π2,π2] [-\frac{\pi}{2}, \frac{\pi}{2}] , which contains one complete positive region.

Step 3: Analyze the function behavior


In the interval [π2,π2] [-\frac{\pi}{2}, \frac{\pi}{2}] , the cosine function is positive, with its maximum value of 1 at x=0 x = 0 . This means our function y=2cos(x) y = 2\cos(x) is non-negative throughout the interval and reaches a maximum value of 2 at x=0 x = 0 .

Step 4: Set up the integral


For a region bounded by y=f(x) y = f(x) , the x-axis, x=a x = a , and x=b x = b , the area is abf(x)dx \int_{a}^{b} f(x) \, dx

For our problem:
Area=π2π22cos(x)dx \text{Area} = \int_{-\frac{\pi}{2}}^{\frac{\pi}{2}} 2\cos(x) \, dx

Step 5: Evaluate the integral



Area=π2π22cos(x)dx \text{Area} = \int_{-\frac{\pi}{2}}^{\frac{\pi}{2}} 2\cos(x) \, dx
=2π2π2cos(x)dx = 2\int_{-\frac{\pi}{2}}^{\frac{\pi}{2}} \cos(x) \, dx
=2sin(x)π2π2 = 2\sin(x) \Big|_{-\frac{\pi}{2}}^{\frac{\pi}{2}}



=2[sin(π2)sin(π2)] = 2[\sin(\frac{\pi}{2}) - \sin(-\frac{\pi}{2})]
=2[1(1)] = 2[1 - (-1)]
=22 = 2 \cdot 2
=4 = 4


Therefore:
Area=4 square units \boxed{\text{Area} = 4 \text{ square units}}
Area=4 square units \boxed{\text{Area} = 4 \text{ square units}}
Question 3
Max Marks: 3

Evaluate 16x2dx \int \sqrt{16 - x^2} \, dx .
integrals
Integrals: Trig Substitution

Tag:

A comprehensive "Filter by Tags" search feature is available on the course homepage, allowing you to find all exercises with this tag.

Prerequisites for this Exercise

No prerequisite skills selected yet.

Evaluate 16x2dx \int \sqrt{16 - x^2} \, dx .

Step 1: Identify the integral type


This is an integral of the form a2x2dx \int \sqrt{a^2 - x^2} \, dx , where a=4 a = 4 . This type of integral can be evaluated using trigonometric substitution.

Step 2: Make the trigonometric substitution


When you have a2x2\sqrt{a^2 - x^2}, substitute x=asinθx = a\sin\theta where π2θπ2-\frac{\pi}{2} \leq \theta \leq \frac{\pi}{2}

Let x=4sinθ x = 4\sin\theta , which gives us:
dx=4cosθdθ dx = 4\cos\theta \, d\theta

For the radical term:

16x2=1616sin2θ \sqrt{16 - x^2} = \sqrt{16 - 16\sin^2\theta}
=16(1sin2θ) = \sqrt{16(1 - \sin^2\theta)}
=16cos2θ = \sqrt{16\cos^2\theta}
=4cosθ=4cosθ = 4|\cos\theta| = 4\cos\theta


Note: We can drop the absolute value signs since cosθ0 \cos\theta \geq 0 in the interval π2θπ2 -\frac{\pi}{2} \leq \theta \leq \frac{\pi}{2} .

Step 3: Substitute and simplify


Substituting into our integral:

16x2dx=4cosθ4cosθdθ \int \sqrt{16 - x^2} \, dx = \int 4\cos\theta \cdot 4\cos\theta \, d\theta
=16cos2θdθ = 16 \int \cos^2\theta \, d\theta


Step 4: Use the half-angle formula


We can use the identity cos2θ=1+cos(2θ)2 \cos^2\theta = \frac{1 + \cos(2\theta)}{2} :


16cos2θdθ=161+cos(2θ)2dθ 16 \int \cos^2\theta \, d\theta = 16 \int \frac{1 + \cos(2\theta)}{2} \, d\theta
=8(1+cos(2θ))dθ = 8 \int (1 + \cos(2\theta)) \, d\theta
=8θ+4sin(2θ)+C = 8\theta + 4\sin(2\theta) + C


Step 5: Convert back to the variable x


We need to express θ \theta and sin(2θ) \sin(2\theta) in terms of x x .

Since x=4sinθ x = 4\sin\theta , we have:
sinθ=x4 \sin\theta = \frac{x}{4}
θ=arcsinx4 \theta = \arcsin\frac{x}{4}

For sin(2θ) \sin(2\theta) , we use the double-angle formula:

sin(2θ)=2sinθcosθ \sin(2\theta) = 2\sin\theta\cos\theta
=2x416x24 = 2 \cdot \frac{x}{4} \cdot \frac{\sqrt{16-x^2}}{4}
=x16x28 = \frac{x\sqrt{16-x^2}}{8}


Substituting back:

8θ+4sin(2θ)+C 8\theta + 4\sin(2\theta) + C
=8arcsinx4+4x16x28+C = 8\arcsin\frac{x}{4} + 4 \cdot \frac{x\sqrt{16-x^2}}{8} + C
=8arcsinx4+x16x22+C = 8\arcsin\frac{x}{4} + \frac{x\sqrt{16-x^2}}{2} + C


Therefore:

16x2dx=8arcsinx4+x16x22+C \boxed{ \begin{array}{c}
\int \sqrt{16 - x^2} \, dx \\
= 8\arcsin\frac{x}{4} + \frac{x\sqrt{16-x^2}}{2} + C
\end{array} }




Prof's perspective


This problem tests your ability to recognize an integral that requires trigonometric substitution

16x2dx=8arcsinx4+x16x22+C \boxed{ \begin{array}{c}
\int \sqrt{16 - x^2} \, dx \\
= 8\arcsin\frac{x}{4} + \frac{x\sqrt{16-x^2}}{2} + C
\end{array} }

Question 4
Max Marks: 3

Evaluate x3x3t3sin(t4)dt \int_{-x^3}^{x^3} t^3 \sin(t^4) \, dt .
integrals
Integrals: Symmetry

Tag:

A comprehensive "Filter by Tags" search feature is available on the course homepage, allowing you to find all exercises with this tag.

Prerequisites for this Exercise

No prerequisite skills selected yet.

Evaluate x3x3t3sin(t4)dt \int_{-x^3}^{x^3} t^3 \sin(t^4) \, dt .

Step 1: Check for symmetry


Always check for symmetry properties first when evaluating definite integrals over intervals like [-a,a]. This can save significant calculation time.

Let's examine if the integrand t3sin(t4) t^3 \sin(t^4) has symmetry properties:
- t4 t^4 is even (same value for t t and t -t )
- sin(t4) \sin(t^4) will have the same value at t t and t -t since t4 t^4 is even
- t3 t^3 is odd (changes sign when t t changes to t -t )

Therefore, t3sin(t4) t^3 \sin(t^4) is an odd function because an even function multiplied by an odd function results in another odd function:
(t)3sin((t)4)=t3sin(t4) (-t)^3 \sin((-t)^4) = -t^3 \sin(t^4)

Step 2: Apply symmetry property of odd functions


For any odd function f(t)=f(t) f(-t) = -f(t) , the definite integral over a symmetric interval [a,a] [-a, a] is always zero:

aaf(t)dt=0 \int_{-a}^{a} f(t) \, dt = 0

Since our integrand is odd and the interval [x3,x3] [-x^3, x^3] is symmetric about the origin, we can immediately conclude:

x3x3t3sin(t4)dt=0 \boxed{\int_{-x^3}^{x^3} t^3 \sin(t^4) \, dt = 0}

Prof's perspective


This problem tests your ability to:

1. Recognize and apply symmetry properties of functions
2. Understand how odd functions behave when integrated over symmetric intervals

The professor included this problem to teach you to always look for structural properties and symmetry before attempting more complicated integration techniques. Recognizing oddness/evenness can save significant time and effort in many calculus problems.
x3x3t3sin(t4)dt=0 \boxed{\int_{-x^3}^{x^3} t^3 \sin(t^4) \, dt = 0}
Question 5
Max Marks: 3

Evaluate 0π/2sin5xcos2xdx \int_{0}^{\pi/2} \sin^5 x \cos^2 x \, dx .
integrals
Integrals: Trig Substitution

Tag:

A comprehensive "Filter by Tags" search feature is available on the course homepage, allowing you to find all exercises with this tag.

Prerequisites for this Exercise

No prerequisite skills selected yet.

Evaluate 0π/2sin5xcos2xdx \int_{0}^{\pi/2} \sin^5 x \cos^2 x \, dx .

Step 1: Identify the integral type


This is an integral involving powers of sine and cosine. We can approach this using substitution or by rewriting in terms of other trigonometric functions.

Step 2: Apply a helpful substitution


For integrals with sinmxcosnx\sin^m x \cos^n x, when at least one exponent is odd, substitute the function with the odd exponent in terms of the other.

Since sin5x\sin^5 x has an odd exponent, let's rewrite it as:
sin5x \sin^5 x
=sin4xsinx=(1cos2x)2sinx = \sin^4 x \cdot \sin x = (1-\cos^2 x)^2 \cdot \sin x
Step 3: Expand and set up substitution


Let's substitute u=cosxu = \cos x, which gives du=sinxdxdu = -\sin x \, dx or sinxdx=du\sin x \, dx = -du.

When x=0x = 0, u=cos(0)=1u = \cos(0) = 1
When x=π/2x = \pi/2, u=cos(π/2)=0u = \cos(\pi/2) = 0

Rewriting our integral:

0π/2sin5xcos2xdx \int_{0}^{\pi/2} \sin^5 x \cos^2 x \, dx
=0π/2(1cos2x)2sinxcos2xdx = \int_{0}^{\pi/2} (1-\cos^2 x)^2 \cdot \sin x \cdot \cos^2 x \, dx
=0π/2(1cos2x)2cos2xsinxdx = \int_{0}^{\pi/2} (1-\cos^2 x)^2 \cdot \cos^2 x \cdot \sin x \, dx
=10(1u2)2u2du = -\int_{1}^{0} (1-u^2)^2 \cdot u^2 \, du
=01(1u2)2u2du = \int_{0}^{1} (1-u^2)^2 \cdot u^2 \, du


Step 4: Expand the expression


Expanding (1u2)2(1-u^2)^2:

(1u2)2=12u2+u4 (1-u^2)^2 = 1 - 2u^2 + u^4


So our integral becomes:

01(12u2+u4)u2du \int_{0}^{1} (1-2u^2+u^4) \cdot u^2 \, du
=01(u22u4+u6)du = \int_{0}^{1} (u^2 - 2u^4 + u^6) \, du
=01u2du201u4du+01u6du = \int_{0}^{1} u^2 \, du - 2\int_{0}^{1} u^4 \, du + \int_{0}^{1} u^6 \, du


Step 5: Evaluate each term



01u2du=u3301=130=13 \int_{0}^{1} u^2 \, du = \left. \frac{u^3}{3} \right|_{0}^{1} = \frac{1}{3} - 0 = \frac{1}{3}

201u4du=2u5501=215=25 -2\int_{0}^{1} u^4 \, du = -2 \left. \frac{u^5}{5} \right|_{0}^{1} = -2 \cdot \frac{1}{5} = -\frac{2}{5}

01u6du=u7701=170=17 \int_{0}^{1} u^6 \, du = \left. \frac{u^7}{7} \right|_{0}^{1} = \frac{1}{7} - 0 = \frac{1}{7}


Step 6: Combine the results



0π/2sin5xcos2xdx=1325+17 \int_{0}^{\pi/2} \sin^5 x \cos^2 x \, dx = \frac{1}{3} - \frac{2}{5} + \frac{1}{7}
=3510542105+15105 = \frac{35}{105} - \frac{42}{105} + \frac{15}{105}
=3542+15105=8105 = \frac{35 - 42 + 15}{105} = \frac{8}{105}


Therefore:

0π/2sin5xcos2xdx=8105 \boxed{ \begin{array}{c}
\int_{0}^{\pi/2} \sin^5 x \cos^2 x \, dx = \frac{8}{105}
\end{array} }



Prof's perspective


This problem tests your ability to:

1. Work with trigonometric integrals involving multiple powers
2. Make appropriate substitutions to simplify complex expressions
3. Use algebraic manipulation to expand expressions strategically

This type of integral frequently appears in physics and engineering applications, particularly in electromagnetic theory and signal processing.

0π/2sin5xcos2xdx=8105 \boxed{ \begin{array}{c}
\int_{0}^{\pi/2} \sin^5 x \cos^2 x \, dx = \frac{8}{105}
\end{array} }

Question 6
Max Marks: 3

Evaluate x2arccos(x3)dx \int x^2 \arccos(x^3) \, dx .
integrals
Integrals: Integration By Parts
integrals: inverse trig
inverse trig functions

Tag:

A comprehensive "Filter by Tags" search feature is available on the course homepage, allowing you to find all exercises with this tag.

Prerequisites for this Exercise

No prerequisite skills selected yet.

Evaluate x2arccos(x3)dx \int x^2 \arccos(x^3) \, dx .

Step 1: Identify the integral type


This is an integral containing a product of a polynomial and an inverse trigonometric function. We can use integration by parts to handle it.

Step 2: Set up integration by parts


When integrating products where one factor is an inverse trigonometric function, choose that function as uu and the remaining factors as dvdv.

Let's use the formula:
udv=uvvdu \int u \, dv = uv - \int v \, du

Let:
- u=arccos(x3) u = \arccos(x^3)
- dv=x2dx dv = x^2 \, dx

Then:
- du=3x21(x3)2dx=3x21x6dx du = \frac{-3x^2}{\sqrt{1-(x^3)^2}} \, dx = \frac{-3x^2}{\sqrt{1-x^6}} \, dx
- v=x2dx=x33 v = \int x^2 \, dx = \frac{x^3}{3}

Step 3: Apply integration by parts



x2arccos(x3)dx \int x^2 \arccos(x^3) \, dx
=x33arccos(x3)x333x21x6dx = \frac{x^3}{3} \arccos(x^3) - \int \frac{x^3}{3} \cdot \frac{-3x^2}{\sqrt{1-x^6}} \, dx
=x33arccos(x3)+x51x6dx = \frac{x^3}{3} \arccos(x^3) + \int \frac{x^5}{\sqrt{1-x^6}} \, dx


Step 4: Evaluate the remaining integral


For the integral x51x6dx \int \frac{x^5}{\sqrt{1-x^6}} \, dx , we can use the substitution u=x6 u = x^6 :


du=6x5dx du = 6x^5 \, dx
x5dx=du6 x^5 \, dx = \frac{du}{6}


This gives us:

x51x6dx=11udu6 \int \frac{x^5}{\sqrt{1-x^6}} \, dx = \int \frac{1}{\sqrt{1-u}} \cdot \frac{du}{6}
=1611udu = \frac{1}{6} \int \frac{1}{\sqrt{1-u}} \, du
=16(2)1u+C = \frac{1}{6} \cdot (-2) \sqrt{1-u} + C
=131x6+C = -\frac{1}{3}\sqrt{1-x^6} + C


Step 5: Substitute back into the original expression



x2arccos(x3)dx \int x^2 \arccos(x^3) \, dx
=x33arccos(x3)+(131x6)+C = \frac{x^3}{3} \arccos(x^3) + \left(-\frac{1}{3}\sqrt{1-x^6}\right) + C
=x33arccos(x3)131x6+C = \frac{x^3}{3} \arccos(x^3) - \frac{1}{3}\sqrt{1-x^6} + C


Therefore:

x2arccos(x3)dx=x33arccos(x3)131x6+C \boxed{ \begin{array}{c}
\int x^2 \arccos(x^3) \, dx \\
= \frac{x^3}{3} \arccos(x^3) - \frac{1}{3}\sqrt{1-x^6} + C
\end{array} }



Prof's perspective


This problem tests your ability to:

1. Handle more complex integration by parts scenarios
2. Recognize the need for multiple substitutions
3. Deal with derivatives of inverse trigonometric functions
4. Manage algebraic complexity while keeping track of all terms

x2arccos(x3)dx=x33arccos(x3)131x6+C \boxed{ \begin{array}{c}
\int x^2 \arccos(x^3) \, dx \\
= \frac{x^3}{3} \arccos(x^3) - \frac{1}{3}\sqrt{1-x^6} + C
\end{array} }

Question 7
Max Marks: 3

Find the area of the region enclosed between y=sin2x y = \sin^2 x and y=cos2x y = \cos^2 x from x=0 x = 0 to x=π2 x = \frac{\pi}{2} .
Formula: Double and Half-Angle
integrals
Find Intersection Points

Tag:

A comprehensive "Filter by Tags" search feature is available on the course homepage, allowing you to find all exercises with this tag.

Prerequisites for this Exercise

No prerequisite skills selected yet.

Find the area of the region enclosed between y=sin2x y = \sin^2 x and y=cos2x y = \cos^2 x from x=0 x = 0 to x=π2 x = \frac{\pi}{2} .

Step 1: Understand the region


We need to find the area between the curves y=sin2x y = \sin^2 x and y=cos2x y = \cos^2 x over the interval [0,π2] [0, \frac{\pi}{2}] .

First, let's evaluate these functions at the endpoints:

At x=0 x = 0 :
sin2(0)=0 \sin^2(0) = 0
cos2(0)=1 \cos^2(0) = 1

At x=π2 x = \frac{\pi}{2} :
sin2(π2)=1 \sin^2(\frac{\pi}{2}) = 1
cos2(π2)=0 \cos^2(\frac{\pi}{2}) = 0

Step 2: Find the intersection point


Let's find where sin2x=cos2x \sin^2 x = \cos^2 x :
sin2x=cos2x \sin^2 x = \cos^2 x

Since sin2x+cos2x=1 \sin^2 x + \cos^2 x = 1 , we have:
sin2x=cos2x=12 \sin^2 x = \cos^2 x = \frac{1}{2}

This occurs when sinx=cosx=12 \sin x = \cos x = \frac{1}{\sqrt{2}} , which happens at x=π4 x = \frac{\pi}{4} .

Step 3: Set up the integral


Based on our analysis:
- For 0<x<π4 0 < x < \frac{\pi}{4} : cos2x>sin2x \cos^2 x > \sin^2 x
- For π4<x<π2 \frac{\pi}{4} < x < \frac{\pi}{2} : sin2x>cos2x \sin^2 x > \cos^2 x

When finding area between curves that cross, split the integral at intersection points and use absolute value of the difference in each region.

Using the tip: the area is calculated as:


Area \text{Area}
=0π4(cos2xsin2x)dx+π4π2(sin2xcos2x)dx = \int_{0}^{\frac{\pi}{4}} (\cos^2 x - \sin^2 x) \, dx + \int_{\frac{\pi}{4}}^{\frac{\pi}{2}} (\sin^2 x - \cos^2 x) \, dx


Step 4: Simplify using trigonometric identities



When working with expressions like sin2xcos2x\sin^2 x - \cos^2 x or cos2xsin2x\cos^2 x - \sin^2 x, always try to apply the double angle formula cos(2x)=cos2xsin2x\cos(2x) = \cos^2 x - \sin^2 x to simplify integration.


We can simplify using the identity cos2xsin2x=cos(2x) \cos^2 x - \sin^2 x = \cos(2x) :


Area=0π4cos(2x)dx+π4π2(cos(2x))dx \text{Area} = \int_{0}^{\frac{\pi}{4}} \cos(2x) \, dx + \int_{\frac{\pi}{4}}^{\frac{\pi}{2}} (-\cos(2x)) \, dx
=0π4cos(2x)dxπ4π2cos(2x)dx = \int_{0}^{\frac{\pi}{4}} \cos(2x) \, dx - \int_{\frac{\pi}{4}}^{\frac{\pi}{2}} \cos(2x) \, dx


Step 5: Evaluate the integrals



cos(2x)dx=12sin(2x)+C \int \cos(2x) \, dx = \frac{1}{2}\sin(2x) + C


0π4cos(2x)dx=12sin(2x)0π4 \int_{0}^{\frac{\pi}{4}} \cos(2x) \, dx = \frac{1}{2}\sin(2x) \Big|_{0}^{\frac{\pi}{4}}
=12sin(π2)12sin(0) = \frac{1}{2}\sin(\frac{\pi}{2}) - \frac{1}{2}\sin(0)
=1210 = \frac{1}{2} \cdot 1 - 0
=12 = \frac{1}{2}

π4π2cos(2x)dx=12sin(2x)π4π2 \int_{\frac{\pi}{4}}^{\frac{\pi}{2}} \cos(2x) \, dx = \frac{1}{2}\sin(2x) \Big|_{\frac{\pi}{4}}^{\frac{\pi}{2}}
=12sin(π)12sin(π2) = \frac{1}{2}\sin(\pi) - \frac{1}{2}\sin(\frac{\pi}{2})
=012 = 0 - \frac{1}{2}
=12 = -\frac{1}{2}



Step 6: Calculate the total area



Total Area=12(12)=12+12=1 \text{Total Area} = \frac{1}{2} - (-\frac{1}{2}) = \frac{1}{2} + \frac{1}{2} = 1


Therefore:

The area between y=sin2x and y=cos2xfrom x=0 to x=π2 is 1 \boxed{ \begin{array}{c}
\text{The area between } y = \sin^2 x \text{ and } y = \cos^2 x \\
\text{from } x = 0 \text{ to } x = \frac{\pi}{2} \text{ is } 1
\end{array} }



Prof's perspective


This problem tests your ability to:

1. Recognize trigonometric identities that simplify the calculation
2. Find intersection points of curves algebraically
3. Set up integrals correctly based on where one curve is above the other
4. Use the double-angle formula for cosine to simplify integration
1 \boxed{ 1}
Question 8
Max Marks: 3

Find the general solution y(t) of the differential equation dydt=t2e3y \frac{dy}{dt} = t^2e^{-3y} .
differential equations: separable
integrals

Tag:

A comprehensive "Filter by Tags" search feature is available on the course homepage, allowing you to find all exercises with this tag.

Prerequisites for this Exercise

No prerequisite skills selected yet.

Find the general solution y(t) of the differential equation dydt=t2e3y \frac{dy}{dt} = t^2e^{-3y} .

Step 1: Recognize the equation as separable


First, I need to identify this as a separable differential equation. The equation can be written with y-terms on one side and t-terms on the other.

For a separable equation dydt=g(t)h(y) \frac{dy}{dt} = g(t)h(y) , rearrange to get 1h(y)dy=g(t)dt \frac{1}{h(y)}dy = g(t)dt and then integrate both sides.

Rewriting the equation:
dydt=t2e3y \frac{dy}{dt} = t^2e^{-3y}

e3ydy=t2dt e^{3y}dy = t^2dt

Step 2: Integrate both sides


Let's integrate both sides:

For the left side:
e3ydy=13e3y+C1 \int e^{3y}dy = \frac{1}{3}e^{3y} + C_1

For the right side:
t2dt=t33+C2 \int t^2dt = \frac{t^3}{3} + C_2

Putting these together with C=C2C1C = C_2 - C_1:
13e3y=t33+C \frac{1}{3}e^{3y} = \frac{t^3}{3} + C

Step 3: Solve for y


Multiply both sides by 3:
e3y=t3+3C e^{3y} = t^3 + 3C

Taking the natural logarithm of both sides:
3y=ln(t3+3C) 3y = \ln(t^3 + 3C)

y=13ln(t3+3C) y = \frac{1}{3}\ln(t^3 + 3C)

We can simplify by setting K = 3C:
y=13ln(t3+K) y = \frac{1}{3}\ln(t^3 + K)

This is our general solution, where K is an arbitrary constant.

y(t)=13ln(t3+K) \boxed{y(t) = \frac{1}{3}\ln(t^3 + K)}

Prof's perspective


Your professor assigned this question to test:
1. Your ability to recognize and solve separable differential equations
2. Your comfort with combining integration and logarithm techniques
3. Your understanding of how to handle constants of integration when dealing with nonlinear equations
y(t)=13ln(t3+K) \boxed{y(t) = \frac{1}{3}\ln(t^3 + K)}
Question 9
Max Marks: 3

Decide whether the following improper integral converges or diverges:
1x2+sinxx3+xlnxdx. \int_{1}^{\infty} \frac{x^2 + \sin x}{x^3 + x\ln x} dx.
Convergence test: ratio
Convergence test: comparison
Convergence test: p-test
logarithms

Tag:

A comprehensive "Filter by Tags" search feature is available on the course homepage, allowing you to find all exercises with this tag.

Prerequisites for this Exercise

No prerequisite skills selected yet.

Step 1: Analyze the integrand's behavior as x approaches infinity


For the improper integral 1x2+sinxx3+xlnxdx \int_{1}^{\infty} \frac{x^2 + \sin x}{x^3 + x\ln x} dx , you need to analyze the behavior of the integrand as x approaches infinity.

First, let's look at the asymptotic behavior by focusing on the highest-degree terms:

x2+sinxx3+xlnxx2x3=1x as x \frac{x^2 + \sin x}{x^3 + x\ln x} \sim \frac{x^2}{x^3} = \frac{1}{x} \text{ as } x \to \infty

When analyzing improper integrals with limits at infinity, compare the integrand to well-known convergent or divergent functions like 1xp \frac{1}{x^p} .

Step 2: Apply the comparison test


Since our integrand behaves like 1x \frac{1}{x} for large values of x, and we know that 11xdx \int_{1}^{\infty} \frac{1}{x} \, dx diverges (it’s the p-integral with p=1 p = 1 ), we use the Comparison Test. We aim to find a positive constant k k such that for sufficiently large x x :

x2+sinxx3+xlnx>kx, \frac{x^2 + \sin x}{x^3 + x \ln x} > \frac{k}{x},

where 1kxdx \int_{1}^{\infty} \frac{k}{x} \, dx diverges, implying our integral diverges if the inequality holds.

Step 3: Find a lower bound


Since 1sinx1 -1 \leq \sin x \leq 1 for all x x , we have:
x21x2+sinxx2+1. x^2 - 1 \leq x^2 + \sin x \leq x^2 + 1.

For x>2 x > 2 , x21>x22 x^2 - 1 > \frac{x^2}{2} , because:
x21>x22 x^2 - 1 > \frac{x^2}{2}
    x2x22>1 \implies x^2 - \frac{x^2}{2} > 1
    x22>1 \implies \frac{x^2}{2} > 1
    x2>2 \implies x^2 > 2
    x>21.414 \implies x > \sqrt{2} \approx 1.414

and x>2 x > 2 satisfies this.

Thus:
x2+sinx>x22. x^2 + \sin x > \frac{x^2}{2}.

For the denominator, since lnx \ln x grows slower than x x , for sufficiently large x x (e.g., x>10 x > 10 ):
x3+xlnx<x3+x3=2x3, x^3 + x \ln x < x^3 + x^3 = 2x^3,


Combining these:
x2+sinxx3+xlnx>x2/22x3=x24x3=14x. \frac{x^2 + \sin x}{x^3 + x \ln x} > \frac{x^2 / 2}{2x^3} = \frac{x^2}{4x^3} = \frac{1}{4x}.

So our lower bound is 14x \frac{1}{4x} .

We gave the rigorous work here, but for only 3 marks you can just state x2+sinx>x22 x^2 + \sin x > \frac{x^2}{2} and x3+xlnx<x3+x3=2x3 x^3 + x \ln x < x^3 + x^3 = 2x^3 without showing any work. But you SHOULD have an intuitive understanding that both of these inequalities are true!

Step 4: Draw the conclusion


Since x2+sinxx3+xlnx>14x \frac{x^2 + \sin x}{x^3 + x\ln x} > \frac{1}{4x} for sufficiently large x, and we know that 114xdx=1411xdx \int_{1}^{\infty} \frac{1}{4x} dx = \frac{1}{4}\int_{1}^{\infty} \frac{1}{x} dx diverges, by the Comparison Test, our original integral also diverges.

The improper integral diverges \boxed{\text{The improper integral diverges}}

Prof's perspective


Your professor assigned this question to test:
1. Your understanding of how to analyze limiting behavior of complex rational functions
2. Your ability to apply comparison tests for improper integrals
3. Your knowledge of which standard improper integrals (like 1xpdx \int \frac{1}{x^p} dx ) converge or diverge


ALTERNATE SOLUTION


Consider the improper integral:
1x2+sinxx3+xlnxdx. \int_{1}^{\infty} \frac{x^2 + \sin x}{x^3 + x \ln x} \, dx.

To determine convergence, analyze the integrand as x x \to \infty . The dominant terms suggest:
x2+sinxx3+xlnxx2x3=1x, \frac{x^2 + \sin x}{x^3 + x \ln x} \sim \frac{x^2}{x^3} = \frac{1}{x},
since sinx \sin x is bounded and xlnx<x2 x \ln x < x^2 for large x x .

We know:
11xdx=limblnx1b \int_{1}^{\infty} \frac{1}{x} \, dx = \lim_{b \to \infty} \ln x \big|_{1}^{b}
=limb(lnbln1) = \lim_{b \to \infty} (\ln b - \ln 1)
=limb(lnb0) = \lim_{b \to \infty} (\ln b - 0)
=limblnb = \lim_{b \to \infty} \ln b
= = \infty

which diverges.

Step 2: Apply the Limit Comparison Test


Let:
- f(x)=x2+sinxx3+xlnx f(x) = \frac{x^2 + \sin x}{x^3 + x \ln x} ,
- g(x)=1x g(x) = \frac{1}{x} .

Both are positive for x>1 x > 1 . Compute the limit:
limxf(x)g(x)=limxx2+sinxx3+xlnx1x \lim_{x \to \infty} \frac{f(x)}{g(x)} = \lim_{x \to \infty} \frac{\frac{x^2 + \sin x}{x^3 + x \ln x}}{\frac{1}{x}}
=limxx2+sinxx3+xlnxx = \lim_{x \to \infty} \frac{x^2 + \sin x}{x^3 + x \ln x} \cdot x
=limxx3+xsinxx3+xlnx = \lim_{x \to \infty} \frac{x^3 + x \sin x}{x^3 + x \ln x}

Factorize:
=limxx3(1+sinxx2)x3(1+lnxx2) = \lim_{x \to \infty} \frac{x^3 (1 + \frac{\sin x}{x^2})}{x^3 (1 + \frac{\ln x}{x^2})}
=limx1+sinxx21+lnxx2 = \lim_{x \to \infty} \frac{1 + \frac{\sin x}{x^2}}{1 + \frac{\ln x}{x^2}}

Evaluate:
- sinxx0 \frac{\sin x}{x} \to 0 as x x \to \infty (since sinx1 |\sin x| \leq 1 ),
- lnxx0 \frac{\ln x}{x} \to 0 as x x \to \infty (since lnx \ln x grows slower than x x ).

Thus:
limx1+sinxx1+lnxx=1+01+0=1. \lim_{x \to \infty} \frac{1 + \frac{\sin x}{x}}{1 + \frac{\ln x}{x}} = \frac{1 + 0}{1 + 0} = 1.

The limit is a positive finite constant.

Step 3: Conclude using the test


By the Limit Comparison Test: if limxf(x)g(x)=c \lim_{x \to \infty} \frac{f(x)}{g(x)} = c where 0<c< 0 < c < \infty , and 1g(x)dx \int_{1}^{\infty} g(x) \, dx diverges, then 1f(x)dx \int_{1}^{\infty} f(x) \, dx diverges. Since 11xdx \int_{1}^{\infty} \frac{1}{x} \, dx diverges and the limit is 1, the original integral diverges.

The improper integral diverges \boxed{\text{The improper integral diverges}}

Prof's perspective


This approach tests:
1. Recognition of asymptotic behavior in rational functions with oscillatory terms,
2. Mastery of the Limit Comparison Test for improper integrals,
3. Familiarity with standard integrals like 11xdx \int_{1}^{\infty} \frac{1}{x} \, dx .
The improper integral diverges \boxed{\text{The improper integral diverges}}
Question 10
Max Marks: 3

Determine whether the following improper integral converges or diverges:
02dx(2x)4/3. \int_{0}^{2} \frac{dx}{(2-x)^{4/3}}.
integrals
Integrals: Improper

Tag:

A comprehensive "Filter by Tags" search feature is available on the course homepage, allowing you to find all exercises with this tag.

Prerequisites for this Exercise

No prerequisite skills selected yet.

Step 1: Identify the improper behavior


For the integral
02dx(2x)4/3 \int_{0}^{2} \frac{dx}{(2-x)^{4/3}}
you need to identify where the integrand becomes improper.

The denominator
(2x)4/3 (2-x)^{4/3}
equals zero when x=2 x = 2 , which is the upper limit of integration. This makes the integral improper since the integrand becomes unbounded at x=2 x = 2 .

When evaluating improper integrals, check if the integrand becomes unbounded at any point in the interval of integration, including the endpoints.

Step 2: Convert to a limit


To handle the improper behavior at x=2 x = 2 , rewrite the integral as a limit:

02dx(2x)4/3 \int_{0}^{2} \frac{dx}{(2-x)^{4/3}}
=limb20bdx(2x)4/3 = \lim_{b \to 2^-} \int_{0}^{b} \frac{dx}{(2-x)^{4/3}}

Step 3: Evaluate the indefinite integral


First, find the indefinite integral:

dx(2x)4/3 \int \frac{dx}{(2-x)^{4/3}}

Use substitution with u=2x u = 2-x , so du=dx du = -dx or dx=du dx = -du :

dx(2x)4/3 \int \frac{dx}{(2-x)^{4/3}}
=duu4/3 = -\int \frac{du}{u^{4/3}}
=u4/3du = -\int u^{-4/3} \, du

Apply the power rule:

u4/3du -\int u^{-4/3} \, du
=u4/3+14/3+1+C = -\frac{u^{-4/3+1}}{-4/3+1} + C
=u1/31/3+C = -\frac{u^{-1/3}}{-1/3} + C
=3u1/3+C = 3u^{-1/3} + C
=3(2x)1/3+C = \frac{3}{(2-x)^{1/3}} + C

Step 4: Evaluate the definite integral as a limit


Now evaluate using the limits:

limb20bdx(2x)4/3 \lim_{b \to 2^-} \int_{0}^{b} \frac{dx}{(2-x)^{4/3}}
=limb2[3(2x)1/3]0b = \lim_{b \to 2^-} \left[ \frac{3}{(2-x)^{1/3}} \right]_{0}^{b}
=limb2(3(2b)1/33(20)1/3) = \lim_{b \to 2^-} \left( \frac{3}{(2-b)^{1/3}} - \frac{3}{(2-0)^{1/3}} \right)
=limb2(3(2b)1/3321/3) = \lim_{b \to 2^-} \left( \frac{3}{(2-b)^{1/3}} - \frac{3}{2^{1/3}} \right)

As b b approaches 2 2 , the term
3(2b)1/3 \frac{3}{(2-b)^{1/3}}
approaches infinity:

limb23(2b)1/3= \lim_{b \to 2^-} \frac{3}{(2-b)^{1/3}} = \infty

Therefore:

limb2(3(2b)1/3321/3) \lim_{b \to 2^-} \left( \frac{3}{(2-b)^{1/3}} - \frac{3}{2^{1/3}} \right)
=321/3 = \infty - \frac{3}{2^{1/3}}
= = \infty

Step 5: Conclusion


Since the limit diverges to infinity, the improper integral diverges:

The improper integral 02dx(2x)4/3 diverges \boxed{\text{The improper integral } \int_{0}^{2} \frac{dx}{(2-x)^{4/3}} \text{ diverges}}
The improper integral 02dx(2x)4/3 diverges \boxed{\text{The improper integral } \int_{0}^{2} \frac{dx}{(2-x)^{4/3}} \text{ diverges}}
Question 11
Max Marks: 3

How many intervals are needed to guarantee that the Simpson's Rule approximation of the integral

I=03g(x)dx I = \int_{0}^{3} g(x) \, dx

lies within

127355 \frac{1}{2^7 \cdot 3^5 \cdot 5}

of the exact value? Assume that

g(4)(x)8for all x obeying 0x3 |g^{(4)}(x)| \leq 8 \quad \text{for all } x \text{ obeying } 0 \leq x \leq 3
Simpson's Rule

Tag:

A comprehensive "Filter by Tags" search feature is available on the course homepage, allowing you to find all exercises with this tag.

Prerequisites for this Exercise

No prerequisite skills selected yet.

Step 1: Recall the error bound for Simpson's Rule


For Simpson's Rule, the error bound is given by:

EK(ba)5180n4 |E| \leq \frac{K(b-a)^5}{180n^4}

where:
- K K is the maximum value of f(4)(x) |f^{(4)}(x)| on [a,b] [a,b] ,
- [a,b] [a,b] is the interval of integration,
- n n is the number of subintervals (must be even).

When working with Simpson's Rule error bounds, try to maintain factors in their original form as you solve for n n , which often leads to cleaner solutions.

Step 2: Identify the given values


From the problem, we know:
- a=0,b=3 a = 0, b = 3 , so ba=3 b-a = 3 ,
- K=8 K = 8 (the maximum value of g(4)(x) |g^{(4)}(x)| ),
- Desired error: E127355 |E| \leq \frac{1}{2^7 \cdot 3^5 \cdot 5} .

Step 3: Set up an inequality for n


We need to find the smallest value of n n (which must be even) such that:

K(ba)5180n4127355 \frac{K(b-a)^5}{180n^4} \leq \frac{1}{2^7 \cdot 3^5 \cdot 5}

Substituting the values:

835180n4127355 \frac{8 \cdot 3^5}{180n^4} \leq \frac{1}{2^7 \cdot 3^5 \cdot 5}

Simplify the left side using the factorization of 180=22325 180 = 2^2 \cdot 3^2 \cdot 5 :

835180n4 \frac{8 \cdot 3^5}{180n^4}

=83522325n4 = \frac{8 \cdot 3^5}{2^2 \cdot 3^2 \cdot 5 \cdot n^4}

=8352225n4 = \frac{8 \cdot 3^{5-2}}{2^2 \cdot 5 \cdot n^4}

=833225n4 = \frac{8 \cdot 3^3}{2^2 \cdot 5 \cdot n^4}

=2333225n4 = \frac{2^3 \cdot 3^3}{2^2 \cdot 5 \cdot n^4}

=2335n4 = \frac{2 \cdot 3^3}{5 \cdot n^4}

So the inequality becomes:

2335n4127355 \frac{2 \cdot 3^3}{5 \cdot n^4} \leq \frac{1}{2^7 \cdot 3^5 \cdot 5}

Multiply both sides by 5n427355 5 \cdot n^4 \cdot 2^7 \cdot 3^5 \cdot 5 :

(233)(27355)5n4 (2 \cdot 3^3) \cdot (2^7 \cdot 3^5 \cdot 5) \leq 5 \cdot n^4

Simplify the left-hand side:

22733355 2 \cdot 2^7 \cdot 3^3 \cdot 3^5 \cdot 5

=21+733+55 = 2^{1+7} \cdot 3^{3+5} \cdot 5

=28385 = 2^8 \cdot 3^8 \cdot 5

So:

283855n4 2^8 \cdot 3^8 \cdot 5 \leq 5 \cdot n^4

Divide by 5:

2838n4 2^8 \cdot 3^8 \leq n^4

Take the fourth root:

n(2838)1/4 n \geq (2^8 \cdot 3^8)^{1/4}

=(28)1/4(38)1/4 = (2^8)^{1/4} \cdot (3^8)^{1/4}

=28/438/4 = 2^{8/4} \cdot 3^{8/4}

=2232 = 2^2 \cdot 3^2

=49 = 4 \cdot 9

=36 = 36

Step 4: Adjust for even n


Simpson's Rule requires n n to be even. Since n36 n \geq 36 , and 36 is even, the smallest even n n satisfying the inequality is:

n=36 n = 36

Step 5: Verify the solution


Substitute n=36 n = 36 :

36=2232 36 = 2^2 \cdot 3^2

364 36^4

=(2232)4 = (2^2 \cdot 3^2)^4

=2838 = 2^8 \cdot 3^8

2335364 \frac{2 \cdot 3^3}{5 \cdot 36^4}

=23352838 = \frac{2 \cdot 3^3}{5 \cdot 2^8 \cdot 3^8}

=252835 = \frac{2}{5 \cdot 2^8 \cdot 3^5}

=152735 = \frac{1}{5 \cdot 2^7 \cdot 3^5}

This matches the desired error bound, confirming n=36 n = 36 is sufficient.

Step 6: Conclusion


The number of intervals needed is:

36 \boxed{36}
36 \boxed{36}
Question 12
Max Marks: 3

Let θn\theta_n be a sequence such that 3<θn<43 < \theta_n < 4 for all nn, and:

n=1tan(θn)=4. \sum_{n=1}^{\infty} \tan(\theta_n) = 4.

Find limnθn\lim_{n \to \infty} \theta_n, or show that the limit does not exist.
Sequences

Tag:

A comprehensive "Filter by Tags" search feature is available on the course homepage, allowing you to find all exercises with this tag.

Prerequisites for this Exercise

No prerequisite skills selected yet.

Step 1: Examine the convergence requirement


For the series n=1tan(θn)\sum_{n=1}^{\infty} \tan(\theta_n) to converge to 4, we must have limntan(θn)=0\lim_{n \to \infty} \tan(\theta_n) = 0.

Otherwise, if tan(θn)\tan(\theta_n) doesn't approach 0, then the terms wouldn't get arbitrarily small, violating a necessary condition for convergence of a series.

Since tan(x)=0\tan(x) = 0 only when x=πx = \pi in our range, this suggests that θn\theta_n must approach π\pi as nn approaches infinity.


limnθn=π \boxed{\lim_{n \to \infty} \theta_n = \pi}
limnθn=π \boxed{\lim_{n \to \infty} \theta_n = \pi}
Question 13
Max Marks: 3

Find the set of all p>0p > 0 for which the following series converges:

n=331(5n)p+2 \sum_{n=33}^{\infty} \frac{1}{\sqrt{(5n)^p + 2}}
Convergence test: p-test

Tag:

A comprehensive "Filter by Tags" search feature is available on the course homepage, allowing you to find all exercises with this tag.

Prerequisites for this Exercise

No prerequisite skills selected yet.

Step 1: Analyze the general term of the series


Since this is an infinite sum, the starting point of n=33n=33 is not different than n=1n=1 or any other starting point.

Let's examine the series:

n=331(5n)p+2 \sum_{n=33}^{\infty} \frac{1}{\sqrt{(5n)^p + 2}}

where we need to find all values of p>0 p > 0 for which this series converges.

Step 2: Compare with a simpler series


For large values of n n , we have (5n)p2 (5n)^p \gg 2 , so:

(5n)p+2(5n)p (5n)^p + 2 \approx (5n)^p

This suggests our series behaves asymptotically like:

n=331(5n)p \sum_{n=33}^{\infty} \frac{1}{\sqrt{(5n)^p}}
=n=331(5n)p/2 = \sum_{n=33}^{\infty} \frac{1}{(5n)^{p/2}}
=15p/2n=331np/2 = \frac{1}{5^{p/2}} \sum_{n=33}^{\infty} \frac{1}{n^{p/2}}

When analyzing series with parameters, focus on the asymptotic behavior for large n and compare with well-known series types.

Step 3: Apply the p-series test


We recognize that n=331np/2 \sum_{n=33}^{\infty} \frac{1}{n^{p/2}} is a p-series with exponent p/2 p/2 .

For a p-series:
- If p/2>1 p/2 > 1 , the series converges
- If p/21 p/2 \leq 1 , the series diverges

Therefore, our approximating series converges when p/2>1 p/2 > 1 or p>2 p > 2 .


Conclusion


The original series n=331(5n)p+2 \sum_{n=33}^{\infty} \frac{1}{\sqrt{(5n)^p + 2}} converges if and only if p>2 p > 2 .

Therefore, the set of all p>0 p > 0 for which the series converges is:

p(2,) \boxed{p \in (2, \infty)}
p(2,) \boxed{p \in (2, \infty)}
Question 14
Max Marks: 3

Find the radius of convergence of the power series

n=0(2n+n2)xn \sum_{n=0}^{\infty}(2^n + n^2)x^n
Power Series: Radius of convergence
Convergence Test: Root

Tag:

A comprehensive "Filter by Tags" search feature is available on the course homepage, allowing you to find all exercises with this tag.

Prerequisites for this Exercise

No prerequisite skills selected yet.

Question


Find the radius of convergence of the power series

n=0(2n+n2)xn \sum_{n=0}^{\infty}(2^n + n^2)x^n

Solution using the root test


Using the root test, we need to find:

L=limn(2n+n2)xn1/n L = \lim_{n \to \infty} \left|(2^n + n^2)x^n\right|^{1/n}

=limn(2n+n2)1/nx = \lim_{n \to \infty} \left|(2^n + n^2)\right|^{1/n} \cdot |x|

=limn(2n+n2)1/nx = \lim_{n \to \infty} (2^n + n^2)^{1/n} \cdot |x|

For large values of n, 2^n grows much faster than n^2, so:

limn(2n+n2)1/n=limn(2n)1/n=2 \lim_{n \to \infty} (2^n + n^2)^{1/n} = \lim_{n \to \infty} (2^n)^{1/n} = 2

Therefore:
L=2x L = 2|x|

For convergence, we need:
2x<1 2|x| < 1
x<12 |x| < \frac{1}{2}

So the radius of convergence is R = 1/2.

Find the interval of convergence


From our calculation, we know the series converges when:
12<x<12 -\frac{1}{2} < x < \frac{1}{2}

Let's check the endpoints:

At x = 1/2:
n=0(2n+n2)(12)n=n=01+n=0n22n \sum_{n=0}^{\infty}(2^n + n^2)\left(\frac{1}{2}\right)^n = \sum_{n=0}^{\infty}1 + \sum_{n=0}^{\infty}\frac{n^2}{2^n}

The first sum diverges.

At x = -1/2:
n=0(2n+n2)(12)n=n=0(1)n+n=0n2(12)n \sum_{n=0}^{\infty}(2^n + n^2)\left(-\frac{1}{2}\right)^n = \sum_{n=0}^{\infty}(-1)^n + \sum_{n=0}^{\infty}n^2\left(-\frac{1}{2}\right)^n

The first sum diverges.

Therefore, the interval of convergence is:
(12,12) (-\frac{1}{2}, \frac{1}{2})

Note: They didn't ask for the interval here, but in case you wanted to see that it's provided for you. The question just asked for the radius, which is:

R=12 \boxed{R = \frac{1}{2}}

R=12 \boxed{R = \frac{1}{2}}
Question 15
Max Marks: 3

Find 1(x+2)(x1)dx. \int \frac{1}{(x+2)(x-1)} \, dx.
Integrals: Partial Fractions
integrals

Tag:

A comprehensive "Filter by Tags" search feature is available on the course homepage, allowing you to find all exercises with this tag.

Prerequisites for this Exercise

No prerequisite skills selected yet.

Step 1: Use partial fraction decomposition


We need to decompose the integrand into simpler fractions. Since we have distinct linear factors in the denominator, we can write:

1(x+2)(x1)=Ax+2+Bx1 \frac{1}{(x+2)(x-1)} = \frac{A}{x+2} + \frac{B}{x-1}

When using partial fractions for expressions with linear factors, create a separate fraction for each factor, then solve for constants by evaluating at strategic points or using the cover-up method.

Step 2: Find the constants A and B


Multiply both sides by (x+2)(x1)(x+2)(x-1):

1=A(x1)+B(x+2) 1 = A(x-1) + B(x+2)
1=AxA+Bx+2B 1 = Ax - A + Bx + 2B
1=(A+B)x+(A+2B) 1 = (A+B)x + (-A+2B)

Comparing coefficients:
- Coefficient of xx: A+B=0A + B = 0
- Constant term: A+2B=1-A + 2B = 1

From the first equation: B=AB = -A
Substituting into the second equation:
A+2(A)=1 -A + 2(-A) = 1
A2A=1 -A - 2A = 1
3A=1 -3A = 1
A=13 A = -\frac{1}{3}

Therefore:
B=A=(13)=13 B = -A = -\left(-\frac{1}{3}\right) = \frac{1}{3}

Step 3: Rewrite the integrand and integrate


Now we can rewrite our integral:

1(x+2)(x1)dx \int \frac{1}{(x+2)(x-1)} \, dx
=(131x+2+131x1)dx = \int \left(-\frac{1}{3} \cdot \frac{1}{x+2} + \frac{1}{3} \cdot \frac{1}{x-1}\right) \, dx
=131x+2dx+131x1dx = -\frac{1}{3} \int \frac{1}{x+2} \, dx + \frac{1}{3} \int \frac{1}{x-1} \, dx

Integrating each term:
13lnx+2+13lnx1+C -\frac{1}{3} \ln|x+2| + \frac{1}{3} \ln|x-1| + C
=13lnx1x+2+C = \frac{1}{3} \ln\left|\frac{x-1}{x+2}\right| + C


Therefore, the integral is:

13lnx1x+2+C \boxed{ \frac{1}{3} \ln\left|\frac{x-1}{x+2}\right| + C}
13lnx1x+2+C \boxed{ \frac{1}{3} \ln\left|\frac{x-1}{x+2}\right| + C}
Question 16
Max Marks: 5

Consider the separable differential equation
dydx=3xy6x6y+12,<x<+. \frac{dy}{dx} = 3xy - 6x - 6y + 12, \quad -\infty < x < +\infty.

(a) Find the general solution. Express your answer in the form y=f(x)y = f(x).
differential equations: separable
logarithms
integrals
Integrals: logarithms
Absolute Value
Exponentials

Tag:

A comprehensive "Filter by Tags" search feature is available on the course homepage, allowing you to find all exercises with this tag.

Prerequisites for this Exercise

No prerequisite skills selected yet.

Step 1: Rearrange the differential equation

The original differential equation is:
dydx=3xy6x6y+12 \frac{dy}{dx} = 3xy - 6x - 6y + 12
This will factor to:
dydx=3(x2)(y2) \frac{dy}{dx} = 3(x - 2)(y - 2)

Step 2: Separate the variables

To separate the variables, we'll rearrange the equation:
dydx=3(x2)(y2) \frac{dy}{dx} = 3(x - 2)(y - 2)
Dividing both sides by (y2)(y - 2):
1y2dydx=3(x2) \frac{1}{y - 2} \frac{dy}{dx} = 3(x - 2)
Now we can separate the variables:
1y2dy=3(x2)dx \frac{1}{y - 2} dy = 3(x - 2) dx

Step 3: Integrate both sides

Integrating the left side:
1y2dy=lny2+C1 \int \frac{1}{y - 2} dy = \ln|y - 2| + C_1
Integrating the right side:
3(x2)dx=3(x2)dx=3(x222x)+C2=3x226x+C2 \begin{align}
\int 3(x - 2) dx &= 3\int (x - 2) dx\\
&= 3\left(\frac{x^2}{2} - 2x\right) + C_2\\
&= \frac{3x^2}{2} - 6x + C_2
\end{align}

Setting these equal:
lny2+C1=3x226x+C2 \ln|y - 2| + C_1 = \frac{3x^2}{2} - 6x + C_2
Combining constants (C3=C2C1C_3 = C_2 - C_1):
lny2=3x226x+C3 \ln|y - 2| = \frac{3x^2}{2} - 6x + C_3

Step 4: Solve for y

Exponentiating both sides:
y2=e3x226x+C3=eC3e3x226x |y - 2| = e^{\frac{3x^2}{2} - 6x + C_3} = e^{C_3} \cdot e^{\frac{3x^2}{2} - 6x}
Let's define C4=eC3C_4 = e^{C_3}, which is a constant:
y2=C4e3x226x |y - 2| = C_4 \cdot e^{\frac{3x^2}{2} - 6x}
Since C4C_4 can be positive or negative, it doesn't matter y2y - 2 is positive or negative, so we can write:
y=2+C4e3x226x y = 2 + C_4 \cdot e^{\frac{3x^2}{2} - 6x}
where C4C_4 is a constant that can be positive or negative.

Step 5: Simplify the exponent


Let's complete the square in the exponent:
3x226x=32(x24x)=32(x24x+44)=32((x2)24)=3(x2)226 \begin{align}
\frac{3x^2}{2} - 6x &= \frac{3}{2}(x^2 - 4x)\\
&= \frac{3}{2}(x^2 - 4x + 4 - 4)\\
&= \frac{3}{2}((x - 2)^2 - 4)\\
&= \frac{3(x - 2)^2}{2} - 6
\end{align}

Now our solution is:
y=2+C4e3(x2)226=2+C4e6e3(x2)22=2+C5e3(x2)22 \begin{align}
y &= 2 + C_4 \cdot e^{\frac{3(x - 2)^2}{2} - 6}\\
&= 2 + C_4 \cdot e^{-6} \cdot e^{\frac{3(x - 2)^2}{2}}\\
&= 2 + C_5 \cdot e^{\frac{3(x - 2)^2}{2}}
\end{align}

where C5=C4e6C_5 = C_4 \cdot e^{-6} is our new constant.

Final Solution

y=2+C5e3(x2)22
\boxed{y = 2 + C_5 e^{\frac{3(x-2)^2}{2}}}

where C5C_5 is an arbitrary constant.

Prof's perspective

The professor designed this question to test:
1. Recognition of separable differential equations - identifying when an equation can be arranged to separate the variables, which is a fundamental skill in differential equations.
2. Completing the square - the technique of rewriting quadratic expressions to highlight the relationship with key points (in this case, x = 2 and y = 2).
y=2+C5e3(x2)22
\boxed{y = 2 + C_5 e^{\frac{3(x-2)^2}{2}}}
Max Marks: 5

(a) Find the general solution. Express your answer in the form y=f(x)y = f(x).
factoring: grouping
logarithms
integrals
differential equations: separable

Tag:

A comprehensive "Filter by Tags" search feature is available on the course homepage, allowing you to find all exercises with this tag.

Prerequisites for this Exercise

No prerequisite skills selected yet.

Step 1: Rearrange the differential equation

The original differential equation is:
dydx=3xy6x6y+12 \frac{dy}{dx} = 3xy - 6x - 6y + 12
This will factor to:
dydx=3(x2)(y2) \frac{dy}{dx} = 3(x - 2)(y - 2)

Step 2: Separate the variables

To separate the variables, we'll rearrange the equation:
dydx=3(x2)(y2) \frac{dy}{dx} = 3(x - 2)(y - 2)
Dividing both sides by (y2)(y - 2):
1y2dydx=3(x2) \frac{1}{y - 2} \frac{dy}{dx} = 3(x - 2)
Now we can separate the variables:
1y2dy=3(x2)dx \frac{1}{y - 2} dy = 3(x - 2) dx

Step 3: Integrate both sides

Integrating the left side:
1y2dy=lny2+C1 \int \frac{1}{y - 2} dy = \ln|y - 2| + C_1
Integrating the right side:
3(x2)dx=3(x2)dx=3(x222x)+C2=3x226x+C2 \begin{align}
\int 3(x - 2) dx &= 3\int (x - 2) dx\\
&= 3\left(\frac{x^2}{2} - 2x\right) + C_2\\
&= \frac{3x^2}{2} - 6x + C_2
\end{align}

Setting these equal:
lny2+C1=3x226x+C2 \ln|y - 2| + C_1 = \frac{3x^2}{2} - 6x + C_2
Combining constants (C3=C2C1C_3 = C_2 - C_1):
lny2=3x226x+C3 \ln|y - 2| = \frac{3x^2}{2} - 6x + C_3

Step 4: Solve for y

Exponentiating both sides:
y2=e3x226x+C3=eC3e3x226x |y - 2| = e^{\frac{3x^2}{2} - 6x + C_3} = e^{C_3} \cdot e^{\frac{3x^2}{2} - 6x}
Let's define C4=eC3C_4 = e^{C_3}, which is a constant:
y2=C4e3x226x |y - 2| = C_4 \cdot e^{\frac{3x^2}{2} - 6x}
Since C4C_4 can be positive or negative, it doesn't matter y2y - 2 is positive or negative, so we can write:
y=2+C4e3x226x y = 2 + C_4 \cdot e^{\frac{3x^2}{2} - 6x}
where C4C_4 is a constant that can be positive or negative.

Final Solution

y=2+C4e3x226x
\boxed{y = 2 + C_4 \cdot e^{\frac{3x^2}{2} - 6x}}

where C4C_4 is an arbitrary constant.
y=2+C4e3x226x
\boxed{y = 2 + C_4 \cdot e^{\frac{3x^2}{2} - 6x}}

where C4C_4 is an arbitrary constant.
Max Marks: 4

(b) A certain solution satisfies y(0)=4y(0) = 4. For this solution, what is the smallest value of y(x)y(x) over all real xx?
differential equations: separable
initial conditions
parabolas
optimization
maximizing and minimizing
Completing the square

Tag:

A comprehensive "Filter by Tags" search feature is available on the course homepage, allowing you to find all exercises with this tag.

Prerequisites for this Exercise

No prerequisite skills selected yet.

We need to find a particular solution that satisfies y(0) = 4, and then determine its minimum value.

Step 1: Start from our general solution


From part (a), we have the general solution:
y=2+C4e3x226x y = 2 + C_4 \cdot e^{\frac{3x^2}{2} - 6x}

To find the minimum value, we first need to simplify the exponent by completing the square.

Step 2: Complete the square in the exponent


We'll rewrite the exponent 3x226x\frac{3x^2}{2} - 6x by completing the square:

3x226x=32(x24x)=32(x24x+44)=32((x2)24)=3(x2)226 \begin{align}
\frac{3x^2}{2} - 6x &= \frac{3}{2}(x^2 - 4x) \\
&= \frac{3}{2}(x^2 - 4x + 4 - 4) \\
&= \frac{3}{2}((x - 2)^2 - 4) \\
&= \frac{3(x - 2)^2}{2} - 6
\end{align}


Completing the square helps us identify where the function reaches its minimum or maximum value, which is essential for optimization problems.

Now our solution becomes:
y=2+C4e3(x2)226=2+C4e6e3(x2)22 y = 2 + C_4 \cdot e^{\frac{3(x - 2)^2}{2} - 6} = 2 + C_4 \cdot e^{-6} \cdot e^{\frac{3(x - 2)^2}{2}}

Let's define a new constant C5=C4e6C_5 = C_4 \cdot e^{-6}:
y=2+C5e3(x2)22 y = 2 + C_5 \cdot e^{\frac{3(x - 2)^2}{2}}

Step 3: Find the specific value of C₅


We're given that y(0) = 4, so we substitute x = 0 into our solution:

4=2+C5e3(02)224=2+C5e3424=2+C5e62=C5e6C5=2e6 \begin{align}
4 &= 2 + C_5 \cdot e^{\frac{3(0 - 2)^2}{2}} \\
4 &= 2 + C_5 \cdot e^{\frac{3 \cdot 4}{2}} \\
4 &= 2 + C_5 \cdot e^{6} \\
2 &= C_5 \cdot e^{6} \\
C_5 &= 2 \cdot e^{-6}
\end{align}


Now we can rewrite our particular solution:
y=2+2e6e3(x2)22=2+2e3(x2)226 y = 2 + 2e^{-6} \cdot e^{\frac{3(x - 2)^2}{2}} = 2 + 2e^{\frac{3(x - 2)^2}{2} - 6}

Step 4: Find the minimum value


To find the minimum value of y, we need to analyze the term 2e3(x2)2262e^{\frac{3(x - 2)^2}{2} - 6}.

Since e3(x2)226e^{\frac{3(x - 2)^2}{2} - 6} is always positive and is multiplied by the positive constant 2, this term is always positive.

The exponential part e3(x2)226e^{\frac{3(x - 2)^2}{2} - 6} reaches its minimum when the exponent 3(x2)226\frac{3(x - 2)^2}{2} - 6 is at its minimum.

The term (x2)2(x - 2)^2 is always non-negative and equals zero when x = 2.

Therefore, the minimum value of 3(x2)226\frac{3(x - 2)^2}{2} - 6 occurs when x = 2, giving:
3(22)226=6 \frac{3(2 - 2)^2}{2} - 6 = -6

So when x = 2, our solution gives:
y=2+2e6 y = 2 + 2e^{-6}

y=2+2e6 \boxed{y = 2 + 2e^{-6}}
y=2+2e6 \boxed{y = 2 + 2e^{-6}}
Question 17

A random variable XX has the following probability density function (PDF):

f(x)={14(x+2),for 2x<0,116(4x),for 0xc,0,otherwise.f(x) = \begin{cases}
\frac{1}{4}(x+2), & \text{for } -2 \leq x < 0, \\
\frac{1}{16}(4-x), & \text{for } 0 \leq x \leq c, \\
0, & \text{otherwise}.
\end{cases}
Piecewise functions
Probabily Density Functions
Probability: Expected Value
Probability: Standard Deviation

Tag:

A comprehensive "Filter by Tags" search feature is available on the course homepage, allowing you to find all exercises with this tag.

Prerequisites for this Exercise

No prerequisite skills selected yet.

Max Marks: 1

17. a) Find cc.
Probabily Density Functions
Piecewise functions

Tag:

A comprehensive "Filter by Tags" search feature is available on the course homepage, allowing you to find all exercises with this tag.

Prerequisites for this Exercise

No prerequisite skills selected yet.

Finding c


For a valid PDF, we need the total probability to equal 1:

f(x)dx=2014(x+2)dx+0c116(4x)dx=1 \begin{align}
\int_{-\infty}^{\infty} f(x) dx &= \int_{-2}^{0} \frac{1}{4}(x+2) dx \\
&+ \int_{0}^{c} \frac{1}{16}(4-x) dx \\
&= 1
\end{align}


First integral:
2014(x+2)dx=14[x22+2x]20=14[0(2)]=12 \begin{align}
\int_{-2}^{0} \frac{1}{4}(x+2) dx &= \frac{1}{4}\left[\frac{x^2}{2} + 2x\right]_{-2}^{0} \\
&= \frac{1}{4}[0 - (-2)] \\
&= \frac{1}{2}
\end{align}


Second integral:
0c116(4x)dx=116[4xx22]0c=116(4cc22) \begin{align}
\int_{0}^{c} \frac{1}{16}(4-x) dx &= \frac{1}{16}\left[4x - \frac{x^2}{2}\right]_{0}^{c} \\
&= \frac{1}{16}\left(4c - \frac{c^2}{2}\right)
\end{align}


Setting the sum equal to 1:
12+116(4cc22)=1116(4cc22)=124cc22=88cc2=16c28c+16=0(c4)2=0 \begin{align}
\frac{1}{2} + \frac{1}{16}\left(4c - \frac{c^2}{2}\right) &= 1 \\
\frac{1}{16}\left(4c - \frac{c^2}{2}\right) &= \frac{1}{2} \\
4c - \frac{c^2}{2} &= 8 \\
8c - c^2 &= 16 \\
c^2 - 8c + 16 &= 0 \\
(c - 4)^2 &= 0
\end{align}


Therefore:
c=4 \boxed{c = 4}
Therefore:
c=4 \boxed{c = 4}
Max Marks: 1

17. b) Find Pr(1X2) Pr(-1 ≤ X ≤ 2) .
Probabily Density Functions

Tag:

A comprehensive "Filter by Tags" search feature is available on the course homepage, allowing you to find all exercises with this tag.

Prerequisites for this Exercise

No prerequisite skills selected yet.

To find this probability, we need to calculate:

Pr(1X2)=12f(x)dx=1014(x+2)dx+02116(4x)dx \begin{align}
&\text{Pr}(-1 \leq X \leq 2) \\
&= \int_{-1}^{2} f(x) dx \\
&= \int_{-1}^{0} \frac{1}{4}(x+2) dx + \int_{0}^{2} \frac{1}{16}(4-x) dx
\end{align}


First integral:
1014(x+2)dx=14[x22+2x]10=14[(0+0)(122)]=14[0(32)]=1432=38 \begin{align}
&\int_{-1}^{0} \frac{1}{4}(x+2) dx \\
&= \frac{1}{4}\left[\frac{x^2}{2} + 2x\right]_{-1}^{0} \\
&= \frac{1}{4}\left[(0 + 0) - \left(\frac{1}{2} - 2\right)\right] \\
&= \frac{1}{4}\left[0 - \left(-\frac{3}{2}\right)\right] \\
&= \frac{1}{4} \cdot \frac{3}{2} \\
&= \frac{3}{8}
\end{align}


Second integral:
02116(4x)dx=116[4xx22]02=116[(82)0]=1166=38 \begin{align}
&\int_{0}^{2} \frac{1}{16}(4-x) dx \\
&= \frac{1}{16}\left[4x - \frac{x^2}{2}\right]_{0}^{2} \\
&= \frac{1}{16}\left[(8 - 2) - 0\right] \\
&= \frac{1}{16} \cdot 6 \\
&= \frac{3}{8}
\end{align}


Therefore:
Pr(1X2)=38+38=68=34 \begin{align}
&\text{Pr}(-1 \leq X \leq 2) \\
&= \frac{3}{8} + \frac{3}{8} \\
&= \frac{6}{8} \\
&= \frac{3}{4}
\end{align}


Pr(1X2)=34 \boxed{\text{Pr}(-1 \leq X \leq 2) = \frac{3}{4}}
Pr(1X2)=34 \boxed{\text{Pr}(-1 \leq X \leq 2) = \frac{3}{4}}
Max Marks: 3

17. c) Find E(X)E(X)
Probability: Expected Value
Piecewise functions
Probabily Density Functions

Tag:

A comprehensive "Filter by Tags" search feature is available on the course homepage, allowing you to find all exercises with this tag.

Prerequisites for this Exercise

No prerequisite skills selected yet.

To find the expected value, we calculate:

E[X] E[X]
=xf(x)dx = \int_{-\infty}^{\infty} x \cdot f(x) dx
=20x14(x+2)dx+04x116(4x)dx = \int_{-2}^{0} x \cdot \frac{1}{4}(x+2) dx + \int_{0}^{4} x \cdot \frac{1}{16}(4-x) dx

First integral:
20x14(x+2)dx \int_{-2}^{0} x \cdot \frac{1}{4}(x+2) dx
=1420(x2+2x)dx = \frac{1}{4}\int_{-2}^{0} (x^2+2x) dx
=14[x33+x2]20 = \frac{1}{4}\left[\frac{x^3}{3} + x^2\right]_{-2}^{0}
=14[0(83+4)] = \frac{1}{4}\left[0 - \left(\frac{-8}{3} + 4\right)\right]
=14[834] = \frac{1}{4}\left[\frac{8}{3} - 4\right]
=148123 = \frac{1}{4} \cdot \frac{8-12}{3}
=1443 = \frac{1}{4} \cdot \frac{-4}{3}
=13 = -\frac{1}{3}

Second integral:
04x116(4x)dx \int_{0}^{4} x \cdot \frac{1}{16}(4-x) dx
=11604(4xx2)dx = \frac{1}{16}\int_{0}^{4} (4x-x^2) dx
=116[2x2x33]04 = \frac{1}{16}\left[2x^2 - \frac{x^3}{3}\right]_{0}^{4}
=116[(32643)0] = \frac{1}{16}\left[\left(32 - \frac{64}{3}\right) - 0\right]
=116[32643] = \frac{1}{16}\left[32 - \frac{64}{3}\right]
=11696643 = \frac{1}{16} \cdot \frac{96-64}{3}
=116323 = \frac{1}{16} \cdot \frac{32}{3}
=23 = \frac{2}{3}

Therefore:
E[X] E[X]
=13+23 = -\frac{1}{3} + \frac{2}{3}
=13 = \frac{1}{3}

E[X]=13 \boxed{E[X] = \frac{1}{3}}
E[X]=13 \boxed{E[X] = \frac{1}{3}}
Max Marks: 4

17. d) Find all values of XX whose distance from E(X) E ( X ) is at most one standard deviation. Write your answer as an interval.

For this part only, your endpoints may be left in calculator-ready form.
Probability: Expected Value
Probabily Density Functions
Probability: Standard Deviation
Key Insight

Tag:

A comprehensive "Filter by Tags" search feature is available on the course homepage, allowing you to find all exercises with this tag.

Prerequisites for this Exercise

No prerequisite skills selected yet.

First, I need to find the standard deviation. For that, I need the variance:

Var(X)=E[X2](E[X])2 Var(X) = E[X^2] - (E[X])^2

I already know E[X]=13E[X] = \frac{1}{3}.

Now I need to find E[X2]E[X^2]:

E[X2] E[X^2]
=20x214(x+2)dx+04x2116(4x)dx = \int_{-2}^{0} x^2 \cdot \frac{1}{4}(x+2) dx + \int_{0}^{4} x^2 \cdot \frac{1}{16}(4-x) dx

First integral:
20x214(x+2)dx \int_{-2}^{0} x^2 \cdot \frac{1}{4}(x+2) dx
=1420(x3+2x2)dx = \frac{1}{4}\int_{-2}^{0} (x^3+2x^2) dx
=14[x44+2x33]20 = \frac{1}{4}\left[\frac{x^4}{4} + \frac{2x^3}{3}\right]_{-2}^{0}
=14[0(164+2(8)3)] = \frac{1}{4}\left[0 - \left(\frac{16}{4} + \frac{2(-8)}{3}\right)\right]
=14[0(4163)] = \frac{1}{4}\left[0 - \left(4 - \frac{16}{3}\right)\right]
=14(1634) = \frac{1}{4} \cdot \left(\frac{16}{3} - 4\right)
=1416123 = \frac{1}{4} \cdot \frac{16-12}{3}
=1443 = \frac{1}{4} \cdot \frac{4}{3}
=13 = \frac{1}{3}

Second integral:
04x2116(4x)dx \int_{0}^{4} x^2 \cdot \frac{1}{16}(4-x) dx
=11604(4x2x3)dx = \frac{1}{16}\int_{0}^{4} (4x^2-x^3) dx
=116[4x33x44]04 = \frac{1}{16}\left[\frac{4x^3}{3} - \frac{x^4}{4}\right]_{0}^{4}
=116[(46432564)0] = \frac{1}{16}\left[\left(\frac{4 \cdot 64}{3} - \frac{256}{4}\right) - 0\right]
=116[256364] = \frac{1}{16}\left[\frac{256}{3} - 64\right]
=1162561923 = \frac{1}{16} \cdot \frac{256-192}{3}
=116643 = \frac{1}{16} \cdot \frac{64}{3}
=43 = \frac{4}{3}

Therefore:
E[X2] E[X^2]
=13+43 = \frac{1}{3} + \frac{4}{3}
=53 = \frac{5}{3}

Now calculate the variance:
Var(X) Var(X)
=E[X2](E[X])2 = E[X^2] - (E[X])^2
=53(13)2 = \frac{5}{3} - \left(\frac{1}{3}\right)^2
=5319 = \frac{5}{3} - \frac{1}{9}
=15919 = \frac{15}{9} - \frac{1}{9}
=149 = \frac{14}{9}

The standard deviation is:
σ=Var(X) \sigma = \sqrt{Var(X)}
=149 = \sqrt{\frac{14}{9}}
=143 = \frac{\sqrt{14}}{3}

Values of X whose distance from E(X) is at most one standard deviation satisfy:
XE(X)σ |X - E(X)| \leq \sigma
X13143 \left|X - \frac{1}{3}\right| \leq \frac{\sqrt{14}}{3}

This means:
143X13143 -\frac{\sqrt{14}}{3} \leq X - \frac{1}{3} \leq \frac{\sqrt{14}}{3}
13143X13+143 \frac{1}{3} - \frac{\sqrt{14}}{3} \leq X \leq \frac{1}{3} + \frac{\sqrt{14}}{3}
1143X1+143 \frac{1-\sqrt{14}}{3} \leq X \leq \frac{1+\sqrt{14}}{3}

1143X1+143 \boxed{\frac{1-\sqrt{14}}{3} \leq X \leq \frac{1+\sqrt{14}}{3}}
1143X1+143 \boxed{\frac{1-\sqrt{14}}{3} \leq X \leq \frac{1+\sqrt{14}}{3}}
Question 18

18. Let f(x)=83xx2. f(x) = \frac{8}{3-x-x^2} .
Series
Series: Geometric
Series: Partial Fractions
Convergence test: ratio
Power Series

Tag:

A comprehensive "Filter by Tags" search feature is available on the course homepage, allowing you to find all exercises with this tag.

Prerequisites for this Exercise

No prerequisite skills selected yet.

Max Marks: 4

No exercise content available.

Series
Series: Geometric
Series: Partial Fractions

Tag:

A comprehensive "Filter by Tags" search feature is available on the course homepage, allowing you to find all exercises with this tag.

Prerequisites for this Exercise

No prerequisite skills selected yet.

Finding the power series expansion of f(x)


Let's begin with the function:
f(x)=83xx2 f(x) = \frac{8}{3-x-x^2}

Step 1: Use partial fractions


I'll find constants A and B such that:
f(x)=A3+x+B1x f(x) = \frac{A}{3+x} + \frac{B}{1-x}

To find these constants, let's first factor the denominator:
3xx2=(3+x)(1x) 3-x-x^2 = (3+x)(1-x)

Now we can write:
8(3+x)(1x)=A3+x+B1x \frac{8}{(3+x)(1-x)} = \frac{A}{3+x} + \frac{B}{1-x}

Multiplying all terms by (3+x)(1-x):
8=A(1x)+B(3+x) 8 = A(1-x) + B(3+x)
8=AAx+3B+Bx 8 = A - Ax + 3B + Bx
8=(A+3B)+(A+B)x 8 = (A + 3B) + (-A + B)x

Comparing coefficients:
A+3B=8 A + 3B = 8
A+B=0 -A + B = 0

From the second equation: A = B
Substituting into the first: A + 3A = 8
Therefore: 4A = 8, so A = 2
And since A = B, we have B = 2

So our partial fraction decomposition is:
f(x)=23+x+21x f(x) = \frac{2}{3+x} + \frac{2}{1-x}
Step 2: Convert to power series


Remember that for the geometric series formula: 11r=n=0rn\frac{1}{1-r} = \sum_{n=0}^{\infty} r^n when r<1|r| < 1. By substituting appropriate values for rr, we can convert rational functions to power series.

For the first term:
23+x \frac{2}{3+x}
=2311+x3 = \frac{2}{3} \cdot \frac{1}{1+\frac{x}{3}}
=2311(x3) = \frac{2}{3} \cdot \frac{1}{1-\left(\frac{-x}{3}\right)}
=23n=0(x3)n = \frac{2}{3} \sum_{n=0}^{\infty} \left(\frac{-x}{3}\right)^n
=n=023(x3)n = \sum_{n=0}^{\infty} \frac{2}{3} \left(\frac{-x}{3}\right)^n

For the second term:
21x \frac{2}{1-x}
=211x = 2 \cdot \frac{1}{1-x}
=2n=0xn = 2 \sum_{n=0}^{\infty} x^n
Step 3: Combine the series


f(x)=n=023(13)nxn+n=02xn f(x) = \sum_{n=0}^{\infty} \frac{2}{3} \left(\frac{-1}{3}\right)^n x^n + \sum_{n=0}^{\infty} 2x^n
f(x)=n=0(2(1)n33n+2)xn f(x) = \sum_{n=0}^{\infty} \left(\frac{2(-1)^n}{3 \cdot 3^n} + 2\right) x^n

Therefore, the sequence of coefficients is:
An=2(1)n33n+2 \boxed{A_n = \frac{2(-1)^n}{3 \cdot 3^n} + 2}
An=2(1)n33n+2 \boxed{A_n = \frac{2(-1)^n}{3 \cdot 3^n} + 2}
Max Marks: 2

18. b) Find the largest value of R for which the power series expansion of f(x)=83xx2f(x) = \frac{8}{3-x-x^2} holds.
Convergence test: ratio
Series
Power Series: Radius of convergence

Tag:

A comprehensive "Filter by Tags" search feature is available on the course homepage, allowing you to find all exercises with this tag.

Prerequisites for this Exercise

No prerequisite skills selected yet.

Solution using the Ratio Test


By the Ratio Test, the interval of convergence is the interval on which limnAn+1Anx<1\lim_{n \to \infty} \left|\frac{A_{n+1}}{A_n}x\right| < 1.

Recall from part (a) that our coefficient formula is:
An=2(1)n33n+2 A_n = \frac{2(-1)^n}{3 \cdot 3^n} + 2

As nn gets large, the first term 2(1)n33n\frac{2(-1)^n}{3 \cdot 3^n} approaches 0 because 13n\frac{1}{3^n} decreases rapidly.

So for large nn:
An2 A_n \approx 2
An+12 A_{n+1} \approx 2

Calculating the limit:
limnAn+1Anx \lim_{n \to \infty} \left|\frac{A_{n+1}}{A_n}x\right|
=limn2+2(1)n+133n+12+2(1)n33nx = \lim_{n \to \infty} \left|\frac{2 + \frac{2(-1)^{n+1}}{3 \cdot 3^{n+1}}}{2 + \frac{2(-1)^n}{3 \cdot 3^n}}x\right|

As nn increases, the terms 2(1)n33n\frac{2(-1)^n}{3 \cdot 3^n} and 2(1)n+133n+1\frac{2(-1)^{n+1}}{3 \cdot 3^{n+1}} become increasingly small because 3n3^n grows exponentially. For large values of nn, these terms approach 0, leaving us with:

2+02+0x \approx \left|\frac{2 + 0}{2 + 0}x\right|
=22x = \left|\frac{2}{2}x\right|
=x = |x|

Which is less than 1 for xx in (1,1)(-1, 1).

Thus, the radius of convergence is 1.

R=1 \boxed{R = 1}

The ratio test works especially well for power series where we have an explicit formula for the coefficients. The radius of convergence is determined by finding where the limit of the ratio is less than 1.
R=1 \boxed{R = 1}
Max Marks: 3

18. (c) A colleague suggests approximating f(n)(0)2(n!)f^{(n)}(0) \approx 2(n!) for large nn. To assess this idea, find f(n)(0)f^{(n)}(0) exactly and then calculate the limit as nn \to \infty for both

(i) the absolute difference, f(n)(0)2(n!)|f^{(n)}(0) - 2(n!)| and

(ii) the relative discrepancy, f(n)(0)2(n!)2(n!)\frac{|f^{(n)}(0) - 2(n!)|}{2(n!)}.
Power Series
Series
limits: general
Sequences

Tag:

A comprehensive "Filter by Tags" search feature is available on the course homepage, allowing you to find all exercises with this tag.

Prerequisites for this Exercise

No prerequisite skills selected yet.

Finding f(n)(0)f^{(n)}(0)


From our power series in part (a):
f(x)=k=0Akxk f(x) = \sum_{k=0}^{\infty} A_k x^k

Where:
Ak=2(1)k33k+2 A_k = \frac{2(-1)^k}{3 \cdot 3^k} + 2

We know that for a power series:
An=f(n)(0)n! A_n = \frac{f^{(n)}(0)}{n!}

Rearranging to find f(n)(0)f^{(n)}(0):
f(n)(0)=Ann! f^{(n)}(0) = A_n \cdot n!
f(n)(0)=(2(1)n33n+2)n! f^{(n)}(0) = \left(\frac{2(-1)^n}{3 \cdot 3^n} + 2\right) \cdot n!
f(n)(0)=2(1)nn!33n+2(n!) f^{(n)}(0) = \frac{2(-1)^n \cdot n!}{3 \cdot 3^n} + 2(n!)

Calculating the absolute difference


For part (i), we need the absolute difference:
f(n)(0)2(n!) |f^{(n)}(0) - 2(n!)|
=2(1)nn!33n+2(n!)2(n!) = \left|\frac{2(-1)^n \cdot n!}{3 \cdot 3^n} + 2(n!) - 2(n!)\right|
=2(1)nn!33n = \left|\frac{2(-1)^n \cdot n!}{3 \cdot 3^n}\right|
=2n!33n = \frac{2 \cdot n!}{3 \cdot 3^n}

To find the limit as nn \to \infty:
limn2n!33n \lim_{n \to \infty} \frac{2 \cdot n!}{3 \cdot 3^n}

Let's analyze how n!3n\frac{n!}{3^n} behaves as nn increases:
n!3n=n(n1)(n2)3213n \frac{n!}{3^n} = \frac{n \cdot (n-1) \cdot (n-2) \cdots 3 \cdot 2 \cdot 1}{3^n}
=n3n13n23332313 = \frac{n}{3} \cdot \frac{n-1}{3} \cdot \frac{n-2}{3} \cdots \frac{3}{3} \cdot \frac{2}{3} \cdot \frac{1}{3}

For large nn, most of these factors exceed 1 (when k3>1\frac{k}{3} > 1, or k>3k > 3).

For n>6n > 6, we can show that:
n!3n>n3111=n3 \frac{n!}{3^n} > \frac{n}{3} \cdot 1 \cdot 1 \cdots 1 = \frac{n}{3}

To demonstrate this more clearly:
n!3n=n(n1)(n2)3213n \frac{n!}{3^n} = \frac{n \cdot (n-1) \cdot (n-2) \cdots 3 \cdot 2 \cdot 1}{3^n}
=n3n13n23332313 = \frac{n}{3} \cdot \frac{n-1}{3} \cdot \frac{n-2}{3} \cdots \frac{3}{3} \cdot \frac{2}{3} \cdot \frac{1}{3}

When n>6n > 6, we have:
- The terms n3\frac{n}{3}, n13\frac{n-1}{3}, n23\frac{n-2}{3}, etc. down to 43\frac{4}{3} are all greater than 1
- Only 33\frac{3}{3}, 23\frac{2}{3}, and 13\frac{1}{3} are less than or equal to 1

To establish the lower bound, we can replace all terms k3\frac{k}{3} where k>3k > 3 with 1 (which makes the product smaller):
n!3n=n3n13n2343332313 \frac{n!}{3^n} = \frac{n}{3} \cdot \frac{n-1}{3} \cdot \frac{n-2}{3} \cdot \ldots \cdot \frac{4}{3} \cdot \frac{3}{3} \cdot \frac{2}{3} \cdot \frac{1}{3}
>n311112313 > \frac{n}{3} \cdot 1 \cdot 1 \cdot \ldots \cdot 1 \cdot 1 \cdot \frac{2}{3} \cdot \frac{1}{3}
=n32313=2n27 = \frac{n}{3} \cdot \frac{2}{3} \cdot \frac{1}{3} = \frac{2n}{27}

And since 2n27>n3\frac{2n}{27} > \frac{n}{3} for n>13.5n > 13.5, we can certainly say that for sufficiently large nn:
n!3n>n3 \frac{n!}{3^n} > \frac{n}{3}

For large values of n, factorial growth (n!) dramatically outpaces exponential growth (a^n) for any fixed base a > 0. This is because n! adds new factors with each increment of n, while a^n merely multiplies by the same factor a. Mathematically, it can be proven that limnn!an=\lim_{n \to \infty} \frac{n!}{a^n} = \infty for any constant a > 0.

Therefore:
limn2n!33n>limn2n33=limn2n9= \lim_{n \to \infty} \frac{2 \cdot n!}{3 \cdot 3^n} > \lim_{n \to \infty} \frac{2 \cdot \frac{n}{3}}{3} = \lim_{n \to \infty} \frac{2n}{9} = \infty

Thus, the absolute difference grows without bound:
limnf(n)(0)2(n!)= \boxed{\lim_{n \to \infty} |f^{(n)}(0) - 2(n!)| = \infty}

Calculating the relative discrepancy


For part (ii), we need the relative discrepancy:
f(n)(0)2(n!)2(n!) \frac{|f^{(n)}(0) - 2(n!)|}{2(n!)}
=2n!33n2(n!) = \frac{\frac{2 \cdot n!}{3 \cdot 3^n}}{2(n!)}
=133n = \frac{1}{3 \cdot 3^n}
=13n+1 = \frac{1}{3^{n+1}}

As nn \to \infty, this approaches 0:
limn13n+1=0 \lim_{n \to \infty} \frac{1}{3^{n+1}} = 0

Therefore:
limnf(n)(0)2(n!)2(n!)=0 \boxed{\lim_{n \to \infty} \frac{|f^{(n)}(0) - 2(n!)|}{2(n!)} = 0}

Conclusion


While the absolute difference between f(n)(0)f^{(n)}(0) and 2(n!)2(n!) grows without bound as nn increases, the relative discrepancy approaches 0. This means that 2(n!)2(n!) becomes an increasingly accurate approximation of f(n)(0)f^{(n)}(0) in relative terms, supporting the colleague's suggestion.
limnf(n)(0)2(n!)= \boxed{\lim_{n \to \infty} |f^{(n)}(0) - 2(n!)| = \infty}

limnf(n)(0)2(n!)2(n!)=0 \boxed{\lim_{n \to \infty} \frac{|f^{(n)}(0) - 2(n!)|}{2(n!)} = 0}
Question 19
Max Marks: 9

For a constant b>0b > 0, consider the plane region bounded by the curves y=bx(1x)y = bx(1-x) and y=bsin(πx)y = b\sin(\pi x) in the first quadrant. Rotating this region around the x-axis produces a solid; let VxV_x denote the resulting volume. Similarly, let VyV_y denote the volume produced by rotating the same region around the y-axis.

Find the value of bb for which Vx(1π112)=VyV_x\left(\frac{1}{\pi} - \frac{1}{12}\right) = V_y.
Find Intersection Points
Volume: Rotating Regions
Volume: Washers Method
Volume: Cylindrical Shells
Integrals: Integration By Parts
integrals

Tag:

A comprehensive "Filter by Tags" search feature is available on the course homepage, allowing you to find all exercises with this tag.

Prerequisites for this Exercise

No prerequisite skills selected yet.

Revised Solution: Finding the value of b


For a constant b>0b > 0, consider the plane region bounded by the curves y=bx(1x)y = bx(1-x) and y=bsin(πx)y = b\sin(\pi x) in the first quadrant.

Step 1: Find the points of intersection


The curves intersect when:
bx(1x)=bsin(πx) bx(1-x) = b\sin(\pi x)
x(1x)=sin(πx) x(1-x) = \sin(\pi x)

The solutions are x=0x = 0 and x=1x = 1 since:
- At x=0x = 0: 0(10)=sin(0)0=00 \cdot (1-0) = \sin(0) \Rightarrow 0 = 0
- At x=1x = 1: 1(11)=sin(π)0=01 \cdot (1-1) = \sin(\pi) \Rightarrow 0 = 0

For problems with intersecting curves, identify where they cross and determine which function has larger values in each subinterval to properly set up the volume integrals.

Step 2: Determine which function is larger


Let's compare f(x)=x(1x)f(x) = x(1-x) and g(x)=sin(πx)g(x) = \sin(\pi x) in the interval [0,1][0,1].

Both functions reach their maximum values at x=0.5x = 0.5:
- f(0.5)=0.50.5=0.25f(0.5) = 0.5 \cdot 0.5 = 0.25
- g(0.5)=sin(π/2)=1g(0.5) = \sin(\pi/2) = 1

Since g(0.5)>f(0.5)g(0.5) > f(0.5), the sine function is always above the parabola in the interval (0,1)(0,1).

Step 3: Calculate VxV_x using the washer method


When rotating around the x-axis, we use the washer method where:
- Outer radius R=bsin(πx)R = b\sin(\pi x) (sine function)
- Inner radius r=bx(1x)r = bx(1-x) (parabola)

Vx=π01[(bsin(πx))2(bx(1x))2]dx V_x = \pi\int_{0}^{1} [(b\sin(\pi x))^2 - (bx(1-x))^2] dx

Vx=πb201[sin2(πx)x2(1x)2]dx V_x = \pi b^2\int_{0}^{1} [\sin^2(\pi x) - x^2(1-x)^2] dx

For the first term, using sin2(πx)=1cos(2πx)2\sin^2(\pi x) = \frac{1-\cos(2\pi x)}{2}:
01sin2(πx)dx=011cos(2πx)2dx \int_{0}^{1} \sin^2(\pi x) dx = \int_{0}^{1} \frac{1-\cos(2\pi x)}{2} dx

=1201dx1201cos(2πx)dx = \frac{1}{2}\int_{0}^{1} dx - \frac{1}{2}\int_{0}^{1} \cos(2\pi x) dx

=121212π[sin(2πx)]01 = \frac{1}{2} - \frac{1}{2} \cdot \frac{1}{2\pi}[\sin(2\pi x)]_{0}^{1}

=120=12 = \frac{1}{2} - 0 = \frac{1}{2}

For the second term:
01x2(1x)2dx=01x2(12x+x2)dx \int_{0}^{1} x^2(1-x)^2 dx = \int_{0}^{1} x^2(1-2x+x^2) dx

=01(x22x3+x4)dx = \int_{0}^{1} (x^2 - 2x^3 + x^4) dx

=[x332x44+x55]01 = \left[\frac{x^3}{3} - \frac{2x^4}{4} + \frac{x^5}{5}\right]_{0}^{1}

=1312+15=1015+630=130 = \frac{1}{3} - \frac{1}{2} + \frac{1}{5} = \frac{10-15+6}{30} = \frac{1}{30}

Therefore:
Vx=πb2(12130) V_x = \pi b^2\left(\frac{1}{2} - \frac{1}{30}\right)

Vx=πb215130 V_x = \pi b^2 \cdot \frac{15-1}{30}

Vx=πb21430=πb2715 V_x = \pi b^2 \cdot \frac{14}{30} = \pi b^2 \cdot \frac{7}{15}

Step 4: Calculate VyV_y using the shell method


When rotating around the y-axis, we use the cylindrical shell method.

For each x from 0 to 1:
- Shell circumference = 2πx\pi x
- Shell height = bsin(πx)bx(1x)b\sin(\pi x) - bx(1-x) (difference between the two curves)
- Shell thickness = dx

Vy=2π01x[bsin(πx)bx(1x)]dx V_y = 2\pi\int_{0}^{1} x \cdot [b\sin(\pi x) - bx(1-x)] dx

Vy=2πb01[xsin(πx)x2(1x)]dx V_y = 2\pi b\int_{0}^{1} [x\sin(\pi x) - x^2(1-x)] dx

Vy=2πb01[xsin(πx)x2+x3]dx V_y = 2\pi b\int_{0}^{1} [x\sin(\pi x) - x^2 + x^3] dx

For the first term, using integration by parts:
01xsin(πx)dx=[xcos(πx)π]01+1π01cos(πx)dx \int_{0}^{1} x\sin(\pi x) dx = \left[-\frac{x\cos(\pi x)}{\pi}\right]_{0}^{1} + \frac{1}{\pi}\int_{0}^{1} \cos(\pi x) dx

=1(1)π0+1π1π[sin(πx)]01 = -\frac{1\cdot(-1)}{\pi} - 0 + \frac{1}{\pi} \cdot \frac{1}{\pi}[\sin(\pi x)]_{0}^{1}

=1π+0=1π = \frac{1}{\pi} + 0 = \frac{1}{\pi}

For the second term:
01x2dx=x3301=13 \int_{0}^{1} -x^2 dx = -\frac{x^3}{3}\bigg|_{0}^{1} = -\frac{1}{3}

For the third term:
01x3dx=x4401=14 \int_{0}^{1} x^3 dx = \frac{x^4}{4}\bigg|_{0}^{1} = \frac{1}{4}

Therefore:
Vy=2πb(1π13+14) V_y = 2\pi b\left(\frac{1}{\pi} - \frac{1}{3} + \frac{1}{4}\right)

Vy=2πb(1π+3412) V_y = 2\pi b\left(\frac{1}{\pi} + \frac{3-4}{12}\right)

Vy=2πb(1π112) V_y = 2\pi b\left(\frac{1}{\pi} - \frac{1}{12}\right)

Step 5: Find the value of b where Vx(1π112)=VyV_x\left(\frac{1}{\pi} - \frac{1}{12}\right) = V_y


We want:
Vx(1π112)=Vy V_x\left(\frac{1}{\pi} - \frac{1}{12}\right) = V_y

πb2715(1π112)=2πb(1π112) \pi b^2 \cdot \frac{7}{15} \cdot \left(\frac{1}{\pi} - \frac{1}{12}\right) = 2\pi b\left(\frac{1}{\pi} - \frac{1}{12}\right)

The term (1π112)\left(\frac{1}{\pi} - \frac{1}{12}\right) appears on both sides, so we can divide by it:

πb2715=2πb \pi b^2 \cdot \frac{7}{15} = 2\pi b

7b215=2b \frac{7b^2}{15} = 2b

7b2=30b 7b^2 = 30b

7b=30 7b = 30

b=307 b = \frac{30}{7}

Therefore:
b=307 \boxed{b = \frac{30}{7}}
b=307 \boxed{b = \frac{30}{7}}
Question 20

Let f(x)=0xttan1(t)dtf(x) = \int_0^x t\tan^{-1}(t) dt. Use this function in parts a and b below.
integrals
Integrals: Integration By Parts
Series
Power Series
inverse trig functions
integrals: inverse trig

Tag:

A comprehensive "Filter by Tags" search feature is available on the course homepage, allowing you to find all exercises with this tag.

Prerequisites for this Exercise

No prerequisite skills selected yet.

Max Marks: 3

20. (a) Solve the integral to produce a simple formula for f(x)f(x).
Integrals: Integration By Parts
integrals

Tag:

A comprehensive "Filter by Tags" search feature is available on the course homepage, allowing you to find all exercises with this tag.

Prerequisites for this Exercise

No prerequisite skills selected yet.

Let f(x)=0xt2etdtf(x) = \int_0^x t^2e^{-t} dt. Use this function in parts a and b below.

(a) Solve the integral to produce a simple formula for f(x)f(x).

Solution to part (a)


To solve the integral
f(x)=0xt2etdt f(x) = \int_0^x t^2e^{-t} dt

I'll use integration by parts with:
u=t2 u = t^2
du=2tdt du = 2t \, dt
dv=etdt dv = e^{-t} dt
v=et v = -e^{-t}

Applying the integration by parts formula:
udv=uvvdu \int u \, dv = uv - \int v \, du

I get:
0xt2etdt=t2(et)0x0x(et)2tdt \int_0^x t^2e^{-t} dt = t^2 \cdot (-e^{-t}) \bigg|_0^x - \int_0^x (-e^{-t}) \cdot 2t \, dt

=x2ex0+20xtetdt = -x^2e^{-x} - 0 + 2\int_0^x te^{-t} dt

Now I'll perform integration by parts again on 0xtetdt\int_0^x te^{-t} dt with:
u=t u = t
du=dt du = dt
dv=etdt dv = e^{-t} dt
v=et v = -e^{-t}

This gives:
0xtetdt=t(et)0x0x(et)dt \int_0^x te^{-t} dt = t \cdot (-e^{-t}) \bigg|_0^x - \int_0^x (-e^{-t}) \cdot dt

=xex0+0xetdt = -xe^{-x} - 0 + \int_0^x e^{-t} dt

=xex+[et]0x = -xe^{-x} + [-e^{-t}]_0^x

=xex+(ex)(e0) = -xe^{-x} + (-e^{-x}) - (-e^{0})

=xexex+1 = -xe^{-x} - e^{-x} + 1

=(x+1)ex+1 = -(x+1)e^{-x} + 1

Substituting this back into our original expression:
f(x)=x2ex+2[(x+1)ex+1] f(x) = -x^2e^{-x} + 2[-(x+1)e^{-x} + 1]

=x2ex2(x+1)ex+2 = -x^2e^{-x} - 2(x+1)e^{-x} + 2

=(x2+2x+2)ex+2 = -(x^2 + 2x + 2)e^{-x} + 2

Therefore:
f(x)=2(x2+2x+2)ex \boxed{f(x) = 2 - (x^2 + 2x + 2)e^{-x}}

When integrating products of polynomials and exponentials, repeated application of integration by parts creates a pattern that eventually terminates, giving a closed-form solution.
f(x)=2(x2+2x+2)ex \boxed{f(x) = 2 - (x^2 + 2x + 2)e^{-x}}
Max Marks: 2

20. (b) Given that the series
S=n=0(cf(n)) S = \sum_{n=0}^{\infty} (c - f(n))
converges, find the constant cc.
Series

Tag:

A comprehensive "Filter by Tags" search feature is available on the course homepage, allowing you to find all exercises with this tag.

Prerequisites for this Exercise

No prerequisite skills selected yet.

Given that the series
S=n=0(cf(n)) S = \sum_{n=0}^{\infty} (c - f(n))
converges, find the constant cc.

I'll start by substituting our formula for f(n)f(n) from part (a):
f(n)=2(n2+2n+2)en f(n) = 2 - (n^2 + 2n + 2)e^{-n}

Therefore:
cf(n)=c[2(n2+2n+2)en] c - f(n) = c - [2 - (n^2 + 2n + 2)e^{-n}]
cf(n)=c2+(n2+2n+2)en c - f(n) = c - 2 + (n^2 + 2n + 2)e^{-n}

For this series to converge, the limit of the general term must approach zero:
limn(cf(n))=0 \lim_{n \to \infty} (c - f(n)) = 0

Taking the limit:
limn[c2+(n2+2n+2)en]=0 \lim_{n \to \infty} [c - 2 + (n^2 + 2n + 2)e^{-n}] = 0

Since limn(n2+2n+2)en=0\lim_{n \to \infty} (n^2 + 2n + 2)e^{-n} = 0 (the exponential decay dominates the polynomial growth), we have:
limn(c2)=0 \lim_{n \to \infty} (c - 2) = 0

Therefore:
c=2 \boxed{c = 2}

For any polynomial function p(x)p(x), no matter how high the degree, the limit limxp(x)ex=0\lim_{x \to \infty} \frac{p(x)}{e^x} = 0. This is because exponential growth will always eventually outpace polynomial growth. This powerful comparison is the key to evaluating many improper integrals and determining convergence of series involving combinations of polynomials and exponentials.
c=2 \boxed{c = 2}
Max Marks: 4

20. (c) Find the sum of the series:
S=n=0(1)n(2n+1)e2n S = \sum_{n=0}^{\infty} (-1)^n (2n+1) e^{-2n}

Do not simplify. Give an exact value.
Power Series
inverse trig functions
Series
Series: Find the Value
Series: Derivatives & Integrals
integrals: inverse trig

Tag:

A comprehensive "Filter by Tags" search feature is available on the course homepage, allowing you to find all exercises with this tag.

Prerequisites for this Exercise

No prerequisite skills selected yet.

Step 1: Identify the approach


Looking at this series, I notice it has the pattern (1)n(-1)^n with terms involving (2n+1)(2n+1), which reminds me of the arctangent series. My strategy here will be to connect this to something we already know well. I'll start by writing out the general form of the series with a variable xx instead of e1e^{-1}, then use what we know about arctangent and its derivatives to find a closed-form expression. Once I have that general formula, I can plug in our specific value of e1e^{-1} to get the final answer. This approach leverages standard calculus techniques to tackle what might otherwise be a tricky summation.

This problem requires knowing the power series representation of arctangent: arctan(x)=n=0(1)nx2n+12n+1\arctan(x) = \sum_{n=0}^{\infty} \frac{(-1)^n x^{2n+1}}{2n+1} and that the derivative of arctangent is ddx[arctan(x)]=11+x2\frac{d}{dx}[\arctan(x)] = \frac{1}{1+x^2}. These fundamental formulas allow us to connect our series to known functions.

Step 2: Connect to the arctangent series


To evaluate this series, I'll relate it to the arctangent function and its derivative. First, recall that the power series for arctangent is:

arctan(x)=n=0(1)nx2n+12n+1 \arctan(x) = \sum_{n=0}^{\infty} \frac{(-1)^n x^{2n+1}}{2n+1}

Taking the derivative of both sides with respect to xx:

ddx[arctan(x)]=n=0ddx[(1)nx2n+12n+1] \frac{d}{dx}[\arctan(x)] = \sum_{n=0}^{\infty} \frac{d}{dx}\left[\frac{(-1)^n x^{2n+1}}{2n+1}\right]

11+x2=n=0(1)nx2n \frac{1}{1+x^2} = \sum_{n=0}^{\infty} (-1)^n x^{2n}

A key technique in evaluating complex series is differentiating or integrating known power series term-by-term. This converts one series into another, often revealing patterns that wouldn't be obvious otherwise.

Step 3: Derive the general formula


Now, if I multiply both sides by xx:

x1+x2=n=0(1)nx2n+1 \frac{x}{1+x^2} = \sum_{n=0}^{\infty} (-1)^n x^{2n+1}

Taking the derivative again:

ddx[x1+x2]=n=0(1)n(2n+1)x2n \frac{d}{dx}\left[\frac{x}{1+x^2}\right] = \sum_{n=0}^{\infty} (-1)^n (2n+1) x^{2n}

1x2(1+x2)2=n=0(1)n(2n+1)x2n \frac{1-x^2}{(1+x^2)^2} = \sum_{n=0}^{\infty} (-1)^n (2n+1) x^{2n}

Step 4: Substitute the specific value


For our series, we substitute x=e1x = e^{-1}:

S=1e2(1+e2)2 \boxed{S = \frac{1-e^{-2}}{(1+e^{-2})^2}}
S=1e2(1+e2)2 \boxed{S = \frac{1-e^{-2}}{(1+e^{-2})^2}}